principles and practice of ophthalmology' pg 2875 2013 q1.all of the ... new anti-anginal drugs...

103
AIPG 2013 Q1.All of the following bones are a part of the inferior wall of the orbit except: 1. Ethmoid 2. Palatine 3. Zygomatic 4. Maxilla Ans. (1) Ethmoid Ref: Albert and Jakobiec's 'Principles and Practice of Ophthalmology' 3/e, pg 2875 The roof (superior wall) of the orbit is formed primarily by the orbital plate of frontal bone and also the lesser wing of sphenoid near the apex of the orbit. The floor (inferior wall) is formed by the orbital surface of maxilla, the orbital surface of zygomatic bone and the minute orbital process of palatine bone. The medial wall is formed primarily by the orbital plate of ethmoid, as well as contributions from the frontal process of maxilla, the lacrimal bone, and a small part of the body of the sphenoid. It is the thinnest wall of the orbit, evidenced by pneumatized ethmoidal cells. The lateral wall is formed by the frontal process of zygomatic and more posteriorly by the orbital plate of the greater wing of sphenoid. The lateral wall is the thickest wall of the orbit, important because it is the most exposed surface, highly vulnerable to blunt force trauma. Q2. Features of iron deficiency anemia are all except: 1. Increased Red cell distribution width (RDW) 2. Decreased serum iron 3. Decreased TIBC 4. Decreased serum ferritin Answer is C (Decreased TIBC): Wintrobes lt/795 Iron deficiency anemia, there is decreased MCV & MCH with an elevated red cell distribution width & increased TIBC, decreased serum iron & decreased serum ferritin. Iron Deficiency Anemia: Decreased MCV & MCH Elevated red cell distribution width Increased TIBC Decreased serum iron Decreased serum ferritin Remember: Low serum iron with increased TIBC is: Iron deficiency anemia * Low serum iron with decreased TIBC is: Anemia of chroniC disease

Upload: phamduong

Post on 23-Mar-2018

213 views

Category:

Documents


1 download

TRANSCRIPT

Page 1: Principles and Practice of Ophthalmology' pg 2875 2013 Q1.All of the ... New Anti-Anginal Drugs Fasudil • Rho kinase inhibitor ... Q7. Which is the most anticoagulant of choice for

AIPG 2013 Q1.All of the following bones are a part of the inferior wall of the orbit except:

1. Ethmoid

2. Palatine

3. Zygomatic

4. Maxilla Ans. (1) Ethmoid

Ref: Albert and Jakobiec's 'Principles and Practice of Ophthalmology'3/e,pg 2875

• The roof (superior wall) of the orbit is formed primarily by the orbital plate of frontal

bone and also the lesser wing of sphenoid near the apex of the orbit.

• The floor (inferior wall) is formed by the orbital surface of maxilla, the orbital surface of

zygomatic bone and the minute orbital process of palatine bone.

• The medial wall is formed primarily by the orbital plate of ethmoid, as well as contributions from the frontal process of maxilla, the lacrimal bone, and a small part of the body of the sphenoid.

• It is the thinnest wall of the orbit, evidenced by pneumatized ethmoidal cells.

• The lateral wall is formed by the frontal process of zygomatic and more posteriorly by the orbital plate of the greater wing of sphenoid.

The lateral wall is the thickest wall of the orbit, important because it is the most exposed surface,

highly vulnerable to blunt force trauma.

Q2. Features of iron deficiency anemia are all except: 1. Increased Red cell distribution width (RDW) 2. Decreased serum iron

3. Decreased TIBC

4. Decreased serum ferritin

Answer is C (Decreased TIBC): Wintrobes lt/795

Iron deficiency anemia, there is decreased MCV & MCH with an elevated red cell distribution width & increased TIBC, decreased serum iron & decreased serum ferritin.

Iron Deficiency Anemia:

• Decreased MCV & MCH

• Elevated red cell distribution width

• Increased TIBC

• Decreased serum iron

• Decreased serum ferritin

Remember:

• Low serum iron with increased TIBC is: Iron deficiency anemia

* Low serum iron with decreased TIBC is: Anemia of chroniC disease

Page 2: Principles and Practice of Ophthalmology' pg 2875 2013 Q1.All of the ... New Anti-Anginal Drugs Fasudil • Rho kinase inhibitor ... Q7. Which is the most anticoagulant of choice for

Q3. Which of the following is a Rho kinase inhibitor?

1. Fasudil

2. Ranolazine

3. Amiloride

4. Nicorandil

Answer is A (Fasudil): Katzung 11̂ /203; Harrison 1̂ /2011

Fasudil is a Rho kinase inhibitor leading to vasodilatation.

Fasudil

Rho kinase (protein kinase) —>Vasocontrictor signal Fasudil is a Rho kinase inhibitor leading to vasodilatation,

New Anti-Anginal Drugs

Fasudil • Rho kinase inhibitor

Trimetazidine • Metabolic modulators

Ivabardine • Direct bradycardic agent

Dedanonoate Protein kinase G facilitator

Glybenclamide Sulphonylureas

Q4. While assessing the efficacy of a newly developed drug in comparison to placebo, the 95% confidence interval in dinical trials is used to check for:

A. Efficacy of the drug

B. Non-efficacy of the drug

C. Both efficacy and non-efficacy of the drug

D. Either efficacy or non-efficacy of the drug

Aswer is A (Efficacy of the drug):

Q5. Tidal volume excessive load is prevented by activation of which following receptors:

a) J receptor

b) Thoracic muscle spindle

c) Bronchial stretch receptors

d) Arterial baroreceptor

Ans is C. Bronchial stretch receptors Ref: Guyton Physiology life p516

Page 3: Principles and Practice of Ophthalmology' pg 2875 2013 Q1.All of the ... New Anti-Anginal Drugs Fasudil • Rho kinase inhibitor ... Q7. Which is the most anticoagulant of choice for

The tidal volume is the volume of air inspired or expired with each normal breath. This

basic rhythm of respiration is generated in the dorsal respiratory group of neurons in the medulla. Thus normal quiet breathing (of tidal volume) is caused by repetitive

signals from the dorsal respiratory group, transmitted mainly to the diaphragm, and expiration results from elastic recoil of the lungs and thoracic cage.

This respiratory signal transmitted to the inspiratory muscles is not an instantaneous

burst of action potentials. Instead, in normal respiration, it begins weakly and increases steadily in a ramp manner for about 2 seconds. Then it ceases abruptly for approx. the

next 3 seconds, which turns off the excitation of the inspiratory muscles and allows elastic recoil of the lungs and the chest wall to cause expiration. Thus this inspiratory

signal is ramps signal and continues in a repetitive fashion to cause normal respiration of the tidal volume.

Bronchial stretch receptors are located in the muscular portions of the walls of the bronchi and bronchioles. These transmit inhibitory signals f through the vagi into the dorsal respiratory

group of neurons when the lungs are overinflated when the lungs become over inflated, the stretch receptors activate an appropriate feedback response that ―switches off‘ the inspiratory

ramp and thus stops further inspiration.This is called the Hering-Breuer inflation reflex.

In human beings, the Hering-Breuer reflex probably is not activated until the tidal volume increases to more than three times normal (greater than about 1.5 liters per

breath). Therefore, this reflex appears to be mainly a protective mechanism for preventing excess lung inflation rather than an important ingredient in normal control

of ventilation

Q6. What changes will occur within 24 hours of use of the thiazide in excretion of?

1. Sodium and Potassium increases, calcium increases

2. Sodium and Potassium decreases, calcium decreases

3. Sodium and Calcium increases, potassium decreases

4. Potassium and Calcium increases sodium decreases

Answer is *a* i.e., Sodium and Potassium increases, calcium increases [Ref: Goodman & Gilman

12/e p. 687; KDT 6/e p. 565-

It is a well known fact that thiazides increase Na+ and K+ excretion and decrease Ca2+

excretion.

But they have asked about acute effect of thiazide on these electrolytes.

Actual effect, of thiazide on Ca2± excretion is variable.

• It has been seen in several trials that urinary calcium excretion increases in the initial

stages of thiazide diuretic therapy.

- The exact mechanism for this is not known but is supposed to be because of its action on distal tubule, which is the site for calcium absorption

- However this increased urinary calcium excretion is clinically insignificant and return

to normal within 2-3 days.

Page 4: Principles and Practice of Ophthalmology' pg 2875 2013 Q1.All of the ... New Anti-Anginal Drugs Fasudil • Rho kinase inhibitor ... Q7. Which is the most anticoagulant of choice for

Q7. Which is the most anticoagulant of choice for coagulation test? 1. 3.2% sodium citrate

2. EDTA

3. Heparin 4. Sodium oxalate

Ans:a *a’ 3.2% sodium citrate [Ref: Dacie and lewis practical hematology 10/e p. 7;

Wintrobe 12/e p. 1]

• The most commonly used anticoagulant (or coagulation samples is sodium citrate in the concentration of 3.2% solution

• For routine tests of blood coagulation whole blood is collected into citrated

anticoagulant using an evactuated sample tube containing a fixed amount of citrate as

anticoagulant in the ratio of one part citrate solution to nine parts of whole blood.

Q8 A baby can breathe and suck at the same time. This is due to:

A. Highly placed larynx

B. Wide short tongue

C. Short soft palate

D. Short pharynx

Answer is A (Highly placed larynx): Morgan Jd/849; Logan Turner l(P/382; Gray's Anatomy 4(fh/584

A baby can suck milk into mouth and because of its palate in mouth is separated from

its nasal cavity so while baby is sucking in milk, can also breath through nose.

When the infant has to swallow, the soft palate rapidly moves upward to dose off the

back of the back of nasal air tube.

At the same time, the epiglottis closes off the larynx and guides the milk into the

esophagus.

Because of these factors, infant can breath and swallow in quick succession.

Q9. A 28 years old female presented with the history of recurrent abortions, pain in calves

for 4 years. Patient is suffering from congenital deficiency of:

A. Protein C

B. Thrombin

C. Plasmin

D. Factor XIII

Answer is A (Protein C): Dutta 168-173

Page 5: Principles and Practice of Ophthalmology' pg 2875 2013 Q1.All of the ... New Anti-Anginal Drugs Fasudil • Rho kinase inhibitor ... Q7. Which is the most anticoagulant of choice for

■Inherited thrombophilia causes both early and late miscarriages due to intravascular thrombosisProtein C. resistance (factor V Leiden mutation) is the, most common cause. Protein

C is the natural inhibitor or coagulation.Dutta (Protein C Deficiency

* Protein C is a vitamin K dependent glycoprotein synthesized by the liver that inactivates

factors Va and Viii a.

Q10. A 22 years old female presented with primary amenorrhea,short stature, widely spaced nipples and webbed

neck. TheKaryotype is:

1. 47, XXY

2. 47, XYY

3. 45, XO

4. 46, XY

Answer is C (45, XO): Shawl4h/96-97

A 22 years old female presented with primary amenorrhea, short widely spaced nipples and webbed neck. The patient

is suffering from Turners syndrome & karyotype is 45, XO.

Q11. Osteoblastic metastasis commonly arise from: Breasts

Prostate Lung

RCC Answer is B (Prostate):

Osteoblasticmetastasiscommonlyarisesfromcarcinomaprostate.

• MC site of primary for bone metastasis: CA ProstateQ

• MC cause of osteoblastic secondaries in males: CA 1Prostate9

| MC cause of osteoblastic secondaries in females; CA Breast ^MC tumor metastize to bone in females: CA Breast

Q12. If a chromosome divides in an axis perpendicular to usual axis of division it is going to form:

1. Ring chromosome

2. Isochromosome 3. Acrocentric chromosome

4. Subtelocentric chromosome Answer is B (Isochromosome): Robbins&h/161 Isochromosome

• Isochromosome formation results when one arm of a chromosomeis lost and the remaining arm is duplicated, resulting inachromosome consisting of two short arms only or of two longarms. '

1 An isochromosome has morphologically identical geneticinformation in both arms.

Q13. Real time polymerase chain reaction is done

1. DNA detection only

2. RNA detection only 3. Both RNA and DNA detection

4. Monitoring amplification of target nucleic acid

Page 6: Principles and Practice of Ophthalmology' pg 2875 2013 Q1.All of the ... New Anti-Anginal Drugs Fasudil • Rho kinase inhibitor ... Q7. Which is the most anticoagulant of choice for

Answer is D (Monitoring amplification of target nucleic ̂ Harper 2&t'/395; Lippincott 3̂ /459 Real time

polymerase chain reaction is done for monitoring amplification of target nucleic acid. Polymerase Chain

Reaction

• PCR is a cell free, test tube method used for amplifying a target sequence of DNA.

• Much faster & more sensitive than cell based cloning

• PCR is used to amplify specific regions of DNA strand (target DNA)

• Most PCR methods typically amplify DNA fragments upto 10

kilobase pairs

Q14. Dental infection from the mandibular first molar spread to the ?

1. Buccal vestibular space

2. Submandibular space

3. Infratemporal space 4. Peterygo mandibular space

[Ans. (A) Ref. Vinod Kapoor 2nd Ed/ Pg 147]

―If the apices of mandibular molar are located above the insertion of buccinator muscle and buccal cortex

eroded it will leads to the formation of the vestibular Abscess. On the other hand if the mandibular molar

apices are located below the insertion of buccinator muscle and buccal cortex is eroded, it will lead to the

buccal space infection.‘‘

Why Not (B).is c o r r e c t [ R e f . Shafer 4th Ed/ Pg 517]

―According to the studies of Tschiasshy, because the root apices of the roots of l8t molar are above the

mylohyoid muscle attachment to the ridge in about 60% of the cases, infection of the sublingual space is most common in cases of the infection of this tooth‖

This mean if option is sublingual then best answer is sublingual not snhmandibular

Q15. The pulp bom that is most likely to be exposed during cavity preparation on deciduous molars

is?

1. Mesiobuccal of first motetr

2. Distobuccal of first molar

3. Mesioobuccal of second molar 4. Distobuccal of second molar

5.

[Ans. (A) Ref. Shobha Tandon 1* Ed/ Pg 289]

―The Mesiobuccal pulp horn of the primary first molars occupies a prominent portion of the pulp

chamber. It is the largest pulp horn in both the maxillary and mandibular first primary molar and is commonly exposed during cavity preparation.‖

Q16.The term Rampant Caries applies to a mouth having ?

1. 5 or more lesions per year

2. 10 or more lesions per year 3. 2 or more lesions per year

4. 12 or more lesions per year

[Ans. (B) Ref. McDonald 5th Ed/ Pg 229]

―The term Rampant Caries should be applied to a caries rate of 10 or more new lesions per year. The distinguishing characteristics of rampant caries are the involvement of proximal surface of lower anterior

teeth and the development of cervical type of caries.**

Page 7: Principles and Practice of Ophthalmology' pg 2875 2013 Q1.All of the ... New Anti-Anginal Drugs Fasudil • Rho kinase inhibitor ... Q7. Which is the most anticoagulant of choice for

Q17. Injuries to Primary teeth occur mostly at which age group ? 1. 6-12 months

2. 3-4 years

3. 5-6 years 4. 1.5-2 .5years

[Ans. (D) Ref. Shobha Tandon 1― Ed Pg 492)

"Children in the age group of 1-2 l/5 years sustain injuries to the primary dentition most

frequently. This is the age when a child learns to toddle and is relatively uncoordinated. 8-11 years children sustain injuries to the permanent dentition more frequently.‖

Q18. The Lorothidol added i n n o n - e u g e n o l paste h a s function of

1. Bacteriostatic

2. Fungicide

3. Accelerator 4. Preservative

Ans. 2

2 Types of periodontal pacts are available. They are ZOE pack and non-eugenol pack.

Non-eugenol pack is based on the reaction between a metallic oxide and fatty-acids. It is supplied in 2

tubes. Composition of non-euqenot pack:

One tube contains zinc oxide, oil for plasticity, a gum for cohesiveness and lorothiodol a fungicide.

The other tube contains liquid coconut fatty acids thickened with rosin and chlorothymol, a bacteriostatic agent.

Q19.Glass ionomer cement is used as a barrier over guttapercha filing before bleaching an endodontkally treated

discolored tooth.

1. To prevent bleaching agent from dissolving the guttapercha. 2. To prevent percolation of the bleaching agent into the apical area

3. To prevent contamination of bleaching agent

4. To prevent discoloration of tooth from obturatingmaterial.

Ans. B

The level of barrier should be 1mm incisal to CEO.

This prevents percolation of bleaching agent into the root canal filling and nearby into the periapex.

This also confines the bleaching agent to the crown thereby preventing cervical root

resorption

Q20. Theodor Croll 1986 is associated with

1. Use of 18% hydrochloric acid and pumice - microabrasion

2. Use of 12 fluted bur to remove the defects

3. Bleaching techniques

Page 8: Principles and Practice of Ophthalmology' pg 2875 2013 Q1.All of the ... New Anti-Anginal Drugs Fasudil • Rho kinase inhibitor ... Q7. Which is the most anticoagulant of choice for

4. Dakin's solution Ans.1(Use of 18% hydrochloric acid and pumice microabrasion)

Croll

Microabrasion technlOMt

18% HCI, pumice and water are mixed

to form a paste to remove the enamel

discolorations

12-fluted carbide bur Used for macroabrasion of discolored teeth

Nutting Combination walking bleach technique: 30% H202 + Sodium perborate

Spasser Walking bleach technique Sodium perborate + water

MC inn's Me inn's technique

5. 5 parts 30% H202 +

6. 5 parts 36% HCU

7. 1 part diethylether

Stewart

Thermocatalytic technique

Haywood

and

Heyman

Niaht auard vital bleaching

10% carbamide peroxide in a tray over- the-counter technique

Reyto

Laser tooth whitening ~

Q21. The optimal flow rate of coolant for sonic and ultrasonic subgingival scaling is

1. 1-11 ml/min

2. 14-23 ml/min

3. 24-32 mt/min

4. 34-40 ml/min

Page 9: Principles and Practice of Ophthalmology' pg 2875 2013 Q1.All of the ... New Anti-Anginal Drugs Fasudil • Rho kinase inhibitor ... Q7. Which is the most anticoagulant of choice for

Acoustic

streaming

It is the unidirectional fluid flow causing by ultrasound waves.

Acoustic

turbulence

This is created when the movement of the tip causes the coolant to accelerate produc

an intensified swirling effect.

Cavitation is the formation of bubbles in water caused by high turbulence. The bubbles implode

produce shock waves throughout the water.

Ans IS* 'B' [Ref. Carranza 10* ed Pg 829J Sonics and ultrasonics contain a water knob, which controlsthe volume of water being

delivered to the insert tip.Water contributes to 3 physiologic effects that enhance the efficacy of

power scalers.

Q22. Ideal root canal filling for primary teeth

1. Caldum hydroxide-iodine mixture 2. Zinc oxide-eugenol

3. Gutta percha

4. Ferric sulphate

Ans.1

ZOE paste 8. Commonly used filling material.

9. Overfilling cause mild foreign body reaction.

10. There is difference between its rate of J resorption and that of tooth root.

Page 10: Principles and Practice of Ophthalmology' pg 2875 2013 Q1.All of the ... New Anti-Anginal Drugs Fasudil • Rho kinase inhibitor ... Q7. Which is the most anticoagulant of choice for

KRI paste • It resorbs rapidly and have no undesirable effects on succedaneous teeth

• Contains bactericidal action

• Contains iodoform camphor.

Maisto paste

• Similar to KRI with additions.

Vitapex • Contains calcium hydroxide + iodoform

• Nearly ideal primary tooth filling material

Guttapercha • Contraindicated, as it is not a resorbable material.

Q23. The eruption of teeth in active and passive phases was defined by:

1. Baer & Benjamin (1974) 2. Gardner (1960)

3. Gottleib and Orban (1933)

4. Massler and Schour (1950) Ans.C

Gottlieb and Orban (1933) elaborated the concept of continuous eruption of the teeth, which they divided into active and passive eruption, the latter being further divided into 4 stages. ‖ — Braz Dent J

About tooth eruption:

11. Tooth eruption is divided into two phases: active and passive eruption o Active eruption: physical movement of tooth from its prefunctional subgingival position to the functional occlusion

• Functional active eruption: is the continued tooth movement due to wear (Gottlieb and Orban, 1933). o Passive eruption is the continued apical movement of the free gingival margin/epithelial attachment, o Occurs after the tooth has reached the functional occlusal plane

12. Garguilo et al (1961) classified and divided passive eruption into four stages:

Q24. During jaw Relations Protrusive Records are used for ?

A. Condylar inclination measurement of one side,

b. Condylar inclination measurement of both sides.

C. Records Centric Relation.

D. Record retruded contact position.

Page 11: Principles and Practice of Ophthalmology' pg 2875 2013 Q1.All of the ... New Anti-Anginal Drugs Fasudil • Rho kinase inhibitor ... Q7. Which is the most anticoagulant of choice for

[Ans. (B) Ref. Winkler 2nd Ed Pg 195]

Protrusive Jaw Relation- A protrusive interocclusal record can register the influence ofthe

condylar paths over themovement of mandible. It enables the condylar guidances of the articulator to be set to an approximation of thepaths of condylar movements in the patient.

Mandibular protrusive movements depend upon and must follow the contour of glenoid fossae, which understandably does not resemble the straight line path of articulator

Q25. The Latency Period after mandibular distraction is ?

1. 5 Days 2. 15 days 3. 1 months

4. 2 months

[Ans. (A) Ref. Vinod Kapoor 2 wEd Pg 561]

Latency Period/ Delay- This period which generally ranges from 4-7 days, allows for the

initial fracture healing to occur, bycallus formation in order to bridge the cut bone segments.

Q26. . CADIA System is used to check ?

1. Progression of Periodontal Disease

2. Furcation Involvement

3. Mobility of Teeth

4. Bleeding Sites [Ans. (A) Ref. Carranza 9th Ed Pg 494 & 10th Ed Pg 587]

Computer-Assisted Densitometric Image Analysis System (CADIA) — In this system, a video camera measures the light transmitted through a radiograph, and the signals from the camera are converted into grey-scale images. This system appears to offer an objective method for

following alveolar bone density changes quantitatively over time. Deas et al usinq replicate measurements of attachment levels and CADIA, demonstrated that

prevalence of progressing lesions in periodontitis

Q27. All of muscle retract the scapula except?

(A) Trapezius

(B)Rhomboid major

(C)Rhomboid minor

(D)Levator Scapulae

Answer is D

―Levator Scapulae is an elevator and downward rotator of scapulae. ―

Levator scapulae is a elevator and downward rotator of scapula

Page 12: Principles and Practice of Ophthalmology' pg 2875 2013 Q1.All of the ... New Anti-Anginal Drugs Fasudil • Rho kinase inhibitor ... Q7. Which is the most anticoagulant of choice for

Movement of scapula

Elevation:Moving the superior border of scapula and the acromion process in

upward direction

Depression:Moving the superior border of scapula and acromion in downward direction

Upward rotation:Moving the scapula so that glenoid so that glenoid cavity faces

upward

Downward Rotation:Moving scapula so that glenoid cavity faces inferiorly

Protaction: Abduction:Moving scapula away from midline

Retraction: Moving the scapula towards the midline Moore Clinical Anatomy, 5th edition Page 756

Q28. ) A woman with infertility receives an ovary transplant from her sister who is an

identical Twin. What type of graft it is?

A. Xenograft

B. Autograft

C. Allograft

D. lsograft

Correct answer : D. lsograft

Q29) Type l hypersensitivity is mediated by which of the following immunoglobulins?

A. IgA

B. IgG

C. IgM

D. IgE

Correct answer : D. IgE

Q30) The serum concentration of which of the following human IgG subclass is

maximum?

A. lgG1

B. IgG2

C. lgG3

D. lgG4

Correct answer : A. lgG1

IgG1 is most common (65%). IgG4 is least common (4%

Page 13: Principles and Practice of Ophthalmology' pg 2875 2013 Q1.All of the ... New Anti-Anginal Drugs Fasudil • Rho kinase inhibitor ... Q7. Which is the most anticoagulant of choice for

Q31) The capsule of Cryptococcus neoformans in a CSF sample is best seen by:

A. Gram stain

B. India ink preparation

C. Giemsa stain

D. Methanamine-Silver stain

Correct answer : B. India ink preparation

With India ink preparation, the capsule can be seen as a clear halo around the cell.

Q32) Which of the following infestation leads to malabsorption?

A. Giardia lamblia

B. Ascaris lumbricoides

C. Necator americanus

D. Ancylostama duodenale

Correct answer : A. Giardia lamblia

Giardiasis can cause fulminant diarrhoea and malabsorption. It may even be asymptomatic

Q33) HIV can be detected and confirmed by:

A. Polymerase Chain Reaction (PCR)

B. Reverse Transcriptase PCR

C. Real Time PCR

D. Mimic PCR

Correct answer : B. Reverse Transcriptase PCR

Q34) A bacterial disease that has been associated with the 3 ―Rs‖ i.e., rats, ricefields,

and rainfall is:

A. Leptospirosis

B. Plague

C. Melioidosis

D. Rodent bite fever

Correct answer : A. Leptospirosis

Rats are important reservoirs. Farmers working in the rice fields are at increased risk of

infection. Disease transmission occurs when rain water contaminated with urine of rats gets

into contact with abraded skin

Q35) With reference to infections with Escherichia coli the following are true except:

A. Enteroaggregative E. coli is associated with Persistant diarrhoea

B. Enterohemorrhagic E. coli can cause haemolytic uraemic syndrome

C. Enteroinvasive E. coli produces a disease similar to salmonellosis

D. Enterotoxigenic E. coli is a common cause of travelers‘ diarrhoea

Correct answer : C. Enteroinvasive E. coli produces a disease similar to salmonellosis

Enteroinvasive E. coli produces a disease similar to shigellosis. (Invasive nature)

Page 14: Principles and Practice of Ophthalmology' pg 2875 2013 Q1.All of the ... New Anti-Anginal Drugs Fasudil • Rho kinase inhibitor ... Q7. Which is the most anticoagulant of choice for

Q36) The following statements are true regarding Clostridium perfringens except:

A. lt is the commonest cause of gas gangrene

B. It is normally present in human faeces

C. The principal toxin of C. perfringens is the alpha toxin

D. Gas gangrene producing strains of C. perfringens produce heat resistant spores

Correct answer : D. Gas gangrene producing strains of C. perfringens produce heat resistant

spores

C. perfringens spores are not resistant to heat. They can be destroyed by boiling.

Q37) All of the following Vibrio sp. are halophilic, except:

A. V. cholerae

B. V. parahaemolyticus

C. V. alginolyticus

D. V. fluvialis

Correct answer : A. V. cholerae

V. cholerae is non halophilic.

Q38) In the small intestine, cholera toxin acts by:

A. ADP-ribosylation of the G regulatory protein

B. Inhibition of adenyl cyclase

C. Activation of GTPase

D. Active absorption of NaCl

Correct answer : A. ADP-ribosylation of the G regulatory protein

Cholera toxin increases adenyl cyclase activity by irreversible ADP ribosylation of GTP binding

domain of adenyl cyclase

Q39) W hich of the following is correct?

1. GENETICS has equal influence on all type of jaw dysplasias.

2. Transverse jaw dysplasias are more genetically determined than other dysplasias. 3. Vertical jaw dysplasias are more genetically determined than other dysplasias.

4. Antero-posterior jaw dysplasias are more genetically determined than others.

[Ans. (c) Ref. Proffit 3rfEd Pg 9,13]

―A result of a survey conducted in U.S. (1989-1994) clearly shows that the vertical jaw dysplasias (open bite, deep bite)are more genetically influenced than other dysplasias. It should also be

noted that in this survey, deep bite was seen to-be more prevalent than open bite.‖

Q40. W hich of the following statement is not correct?

1. Bone destruction in periodontal disease is caused by local factors.

2. All gingivitis does not progress to periodontitis.

3. Periodontitis is always preceded by gingivitis. 4. Level of periodontal bone reflects the present inflammatory condition.

Page 15: Principles and Practice of Ophthalmology' pg 2875 2013 Q1.All of the ... New Anti-Anginal Drugs Fasudil • Rho kinase inhibitor ... Q7. Which is the most anticoagulant of choice for

(Ans. (d) Ref. Carranza 9lb Ed/ Pg 487]

A clinical diagnosis of periodontitis is made by measuring the loss of connective tissue

attachment to the root surface (clinical attachment loss) and loss of alveolar bone (radiographic bone loss). This information provides evidence of past periodontal destruction, as well as its extent and severity.‖

Q41. Adenomatoidodonotogenic tumor is also known as

1. Pindborgs tumor

2. Adenoameloblastoma

3. Adenoid cystic carcinoma

4. Ameloblastic fibroma [Ans (b) Ref: - Shafer^* Ed., page 289]

Q42. The difference in the colour between sub and supragingival calculus is related to ?

A. PH of plaque

B. Death of leucocytes C. Hemolysis of erythrocytes

D. All of the above [Ref. Carranza 9th Ed Pg 98] [Ans. (C)]

The inorganic component of plaque is predominantly calcium and phosphorus, with trace amounts of other minerals such as sodium, potassium, and fluoride. The source of

inorganic constituents of supragingival plaque is primarily saliva; as the mineral content increases, plaque mass calcifies to form calculus. Calculus is frequently found in areas

of the dentition adjacent to salivary ducts (e.g., the lingual surface of mandibular anteriors and buccal surface of the maxillary first molars), reflecting the high concentration of minerals available from saliva in those regions.

The inorganic component of subgingival plaque is derived from crevicular fluid, which is a serum transudate. Calcification of subgingival plaque also results in calculus

formation. Subgingival calculus is typically dark green or dark brown, probably reflecting the presence of subgingival matrix components distinct from those of supragingival calculus (e.g., blood products associated with subgingival hemorrhage).

Q43. Which is characterstic of supragingival plaque and not of subgingival plaque in humans ?

1. Motile bacteria are predominant

2. Spirochetes are evident microscopically 3. Gram negative bacteria are predominant

4. Bacterial composition is altered by dietary sugar composition [Ref. Carranza 8th Ed Pg 85 & 9th Ed Pg 98]

[Ans <D)]

Q44. The treatment of choice in comminutted fracture mandible is ?

1. 2.5 mm plate

2. Dynamic compression plate with eccentric hole

3. Reconstruction plate with central holes

4. 1.5mm plate [Ref. ORAL AND MAXILLOFACIAL TRAUMA by Raymond J. Fonseca 3rd Ed, Vol. 2,

Pg 1146]

[Ans.(D)]

Page 16: Principles and Practice of Ophthalmology' pg 2875 2013 Q1.All of the ... New Anti-Anginal Drugs Fasudil • Rho kinase inhibitor ... Q7. Which is the most anticoagulant of choice for

The traditional reconstruction fixation plate requires prevent adaptation to the bone to prevent displacements of fracture segments.

The advantage of reconstruction plate is that it is a load bearing plate. If bone loss,

severe comminution, or gross instability are present across the fracture, the reconstruction plate will provide stability and support the fracture segments during

function as the fracture heals.

Q45. The modulus of elasticity is defined as original shape is evaluated by a) Brittleness

b) Resilience

c) Tensile strength

d) Toughness

Ans.B

Resilience is the amount of energy absorbed by a structure when it is stressed to

proportional limit. The

clastic area of the stress-sttain graph gives resilience; entire area up to breaking point is a measure of toughness. The slope of straight line gives Young's modulus.

Q46.Brinnel hardness number of a dental gold alloy is

1. Tensile strength

2. Elongation 3. Modulus of elasticity 4. Modulus of resilence

Ans. A

Brinnel and Rockwell tests are classified as macro hardness fesfs and they are not

suitable for brittle

materials. The Knoop and Vickers tests are classified as micro hardness tests. Both of

these tests employ loads less

than 9.8H.

The Shore and the Barcol tests are used for measuring the hardness of rubbers and plastics.

The Brinnel test is the one of oldest tests used for determining the hardness of

materials and is directly related to proportional limit and the ultimate tensile strength of dental gold alloys.

The convenience of the Rockwell test, with direct reading of the depth of the indentation, has lead to its wide usage.

The Knoop hardness test is used to obtain the values for both exceedingly hard and soft materials. The hardness value is independent of the ductility of the material.

The Vickers test is employed for dental casting gold alloys. It is suitable for determining the hardness of brittle materials.

Q47. The wetting of an adherent surface by an adhesive is related to: a) Surface energy of the adherent

b) Surface texture ol the adherent

c) Surface tension of the adherent

Page 17: Principles and Practice of Ophthalmology' pg 2875 2013 Q1.All of the ... New Anti-Anginal Drugs Fasudil • Rho kinase inhibitor ... Q7. Which is the most anticoagulant of choice for

d) Nature of the adherent that is crystalline or amorphous Ans: 'A '[R e f. PHILLIPS'11th ed Pg 37]

The property of wetting and adhesions are directly related to surface energy of

adhesives. Substances with low surface energy like Teflon are often used to prevent the

adhesion of films to a surface. Metals, because of their high surface energy, interact

vigorously with liquid adhesives. The surface energy of many restorative materials is

higher when compared to that of tooth. So there is greater tendency for the surface and margins cf restorations to accumulate debris. This property will explain the relatively

high incidence of marginal caries seen around dental restorations

Q48. All Class 111 lesions should be filled with composite resin, because they are

esthetically important.

1. Both the statement and the reason are correct and related

2. Both the statement and the reason are correct but NOT related

3. The statement is correct, but the reason is NOT

4. NEITHER the statement NOR the reason is correct

Ans.4 Think closely, distolingual Class III lesions are relatively nonesthetic and consider the best

materia) whether it be amalgam or direct gold.

Composite resin is not recommended for Class III lesions on the distal-lingual aspect of canines (use either amalgam or direct gold). Composite material will not maintain the

mesiodistal dimension of the tooth. Note: This may not be entirely true today due to the fact that there are much better wear resistant resins than in the past, however, for questions

composite is not recommended for this situation.

A lingual approach is made when preparing a Class III dental amalgam preparation for the

distal of a canine because a lingual approach preserves the esthetic value of the facial surface.

Remember:

A bite-wing radiograph is the best method to diagnose incipient carious lesions on the distal surface of canine teeth.

A diagnostic aid to be used as a last resort to confirm the presence of a carious lesion on the proximal surface of an anterior tooth is mechanical separation (usually with a wedge).

Rule of thumb: When two teeth adjacent to each other have Class III lesions, you should prepare the larger one first and fill the smaller one first. Access to the preps and

shade matching are easier when you do both at the same appointment. Q49.The ideal amount of dentin required between an amalgam restoration and the pulp for

insulation is:

1. 0.5 mm

2. 1.0 mm 3. 2.0 mm 4. 3.0 mm

Ans.C 2.1 mm (1.0 -1.5 mm is acceptable but not idea!)

*** Amalgam is a poor thermal insulator; this is why a base of either calcium hydroxide or zinc oxide eugenol is placed under most amalgam restorations (toprovide thermal protection). Comparison of Restorative Materials

Characteristic Direct Gold Amalgam Composi te

Close adaptation to cavity walls Very good Good Good Coefficient of thermal expansion similar to tooth structure

Very good Good Poor Chemically acceptable to hard and soft tissues

Very good Good Poor

Page 18: Principles and Practice of Ophthalmology' pg 2875 2013 Q1.All of the ... New Anti-Anginal Drugs Fasudil • Rho kinase inhibitor ... Q7. Which is the most anticoagulant of choice for

Absence of irritation to gingival Very good Good Poor

High edge strength Very good Poor Poor

Non-corroding in oral fluids Very good Fair N/A

Insolubility in oral fluids Excellent Excellent Good

Clinical longevity Very good Good Fair

High abrasion resistance Good Good Fair

Permanently restores M-D dimension Good Very good Fair

Withstands masticatory forces Fair Good Poor

Versatility in use Fair Good Poor

Imitates natural tooth color Fair Poor Very good

Ease of manipulation and placement Poor Good Very good

Thermal and elecrrical insulator Very good Poor Good

Q50. Which tooth requires special attention when preparing the occlusal aspect for a

restoration?

1. Mandibular first bicuspid 2. Mandibular second bicuspid 3. Maxillary first molar 4. Maxillary first bicuspid

Ans.4

The key to this question is the angulation of the preparation. The bur should be tilted

lingually to prevent encroachment on the facial pulp horn and also to maintain dentinal support of the lingual cusp. The pulpal floor should be parallel to the occlusal plane of the tooth faciolingually.

Note: Pulpal floor slopes to coincide with the slope (height) of the cusps.

Remember: The area of the tooth that is most sensitive during cavity preparation is DEJ.

Q51. All of the following statements are true regarding glass ionomer restorations

EXCEPT one. Which one is the EXCEPTION?

1. Glass ionomer is often the ideal material of choice for restoring root surface caries in

patients with high caries activity

2. The best surface finish for a glass ionomer restoration is that obtained against a surface

matrix 3. Glass ionomer adheres to mineralized tooth tissue

4. 1Glass ionomers are somewhat esthetic and polish much better than composites

Ans.4

It is true that glass ionomers are somewhat esthetic, however, they do not polish as well as

composites. Both self-cured and light-cured versions of glass ionomers are available. Light-cured glass ionomers are preferred because of both the extended working time and their improved

physical properties. Because of their limited strength and wear resistance, glass ionomers

are indicated generally for the restoration of low stress areas where caries ac- tivity

potential is of significant concern.

Compared to composites, glass ionomers:

Have a lower compressive strength, tensile strength, and hardness Are generally very technique sensitive because of their high solubility when first mixed

Note: With the newer hybrid or light-cured resin-modified glass ionomers, the above

Page 19: Principles and Practice of Ophthalmology' pg 2875 2013 Q1.All of the ... New Anti-Anginal Drugs Fasudil • Rho kinase inhibitor ... Q7. Which is the most anticoagulant of choice for

properties have been improved. Glass ionomers are generally considered the nearly ideal base/liner material because of

the following properties:

Adhesive bond to tooth structure Snap set in the light-cured fonn (for example, Vitrebond) Anticariogenic: due to fluoride release

Bond to composite: makes for excellent liners for Class V root caries restorations. 1Sometimes called the "sandwich technique". This technique achieves all the benefits of

the glass ionomer cements plus the high polishability, surface hardness, and strong bond

to enamel of the composite resin.

Q52. When comparing the physical properties of filled resins to unfilled resins, all of the

following are true EXCEPT one. Which one is the EXCEPTION!

1. Filled resins are harder

2. Unfilled resins have a higher coefficient of thermal expansion 3. Filled resins have a higher compressive strength

4. Filled resins have a lower tensile strength

Ans.4

This is false; filled resins have a higher tensile strength. The most common classification method for composite resins is based on filler content. Filler

particle size, and the method of filler addition. Almost all important properties of composite resins are improved by using higher filler levels. However, as the filler level is increased, the

fluidity decreases.

Highly filled resins typically contain large filler particles but this composition results in a rough finished surface. Smaller filler particles are used to produce a resin that has a relatively

smooth finished surface. Resin filler particles are called:

Macrofillers: 10-100 microns in diameter

Midifillers: 1-10 microns in diameter

Minifillers: 0.1-1 microns in diameter Microfillers: 0.04-0.1 micron in diameter. Examples include Denta-colour, Durafill, Heliomolar RO. and Silux Plus.

*** Hybrid resins contain a mixture of particles with different diameters which allows higher

filler levels and still permits good finishing. The principal particle size is in the 1 to 3 Jim Charisma, Herculite XRV, Prodigy, Tetric, TPH (Total Performance Hybrid), and Z-100 are ex-

amples. Note: Hybrid and microfill resins utilize colloidal silica fillers which are useful for increasing

the hardness and wear resistance of the base resin material while maintaining high pol-

ishability and overall esthetic qualities.

*** New resins with nanofillers that range in size from .005 to 0,01 micron have recently been

developed. These particles are so small that very high filler levels can be achieved while still maintaining workable consistencies. Supreme H-NF and Simile H-NF are examples.

Q53. There are several types of bleaching products available for use at home, which can either

be dispensed by a dentist or purchased over-the-counter.Currently, only dentist- dispensed

home-use tray-appliedgels carry the ADA Seal.

1. 30% carbamide peroxide

2. 15% hydrogen peroxide 3. 20% hydrogen peroxide

Page 20: Principles and Practice of Ophthalmology' pg 2875 2013 Q1.All of the ... New Anti-Anginal Drugs Fasudil • Rho kinase inhibitor ... Q7. Which is the most anticoagulant of choice for

4. 10% carbamide peroxide Ans.4

Two methods of bleaching:

―In office‖: most use a light-activated solution of 35% peroxide in 4-10 minute cycles. This procedure is called ―chairside bleaching‖ and may require more than one office visit. Each visit

may take from 30 minutes to one hour. Note: Lasers have been used during tooth whitening procedures to enhance the action of the whitening

agent.

I "At home‖: the active ingredient contained in all of the at home tooth whiteners which have earned the ADA's seal, and the compound which has been evaluated in the vast majority of at

home bleaching studies, is carbamide peroxide at a concentration of 10%. The active ingredient

found in most over-the-counter at home bleaching products is not carbamide peroxide but

instead hydrogen peroxide.

Note: Bleaching can affect the color of dentin and enamel. Extrinsic stains respond best to

vital beaching. Response is best with yellow stain followed by brown and orange. The worst response is from gray stains (tetracycline staining).

Other ways to lighten vital teeth:

• Direct composites: useful for tetracycline staining

• Laboratory-fabricated porcclain veneers: useful when the shape, size and arrange-

ment of teeth are esthetically unacceptable

• Full-coverage crowns (most invasive and costly)', may be all-ceramic or porcelain fused

to metal

Q54. All of the following are a part of the innate immunity except? (A).

Complement system

(B). NK cells

(C).Macrophages

(D).T Cells

Ans. (D). T Cells

Innate Immunity

The human body has the ability to resist almost all types of organisms or toxins that

tend to damage the tissues and organs. This capability is called immunity.

Much of immunity is acquired immunity that does not develop until after the body is

first attacked by a bacterium, virus, or toxin, often requiring weeks or months to

develop the immunity. An additional portion of immunity results from general

processes, rather than from processes directed at specific disease organisms. This is

called innate immunity.

It includes the following:

1. Phagocytosis of bacteria and other invaders by white blood cells and cells of the tissue

macrophage system.

2. Destruction of swallowed organisms by the acid secretions of the stomach and the

digestive enzymes.

3. Resistance of the skin to invasion by organisms.

4. Presence in the blood of certain chemical compounds that attach toforeign organisms

or toxins and destroy

Page 21: Principles and Practice of Ophthalmology' pg 2875 2013 Q1.All of the ... New Anti-Anginal Drugs Fasudil • Rho kinase inhibitor ... Q7. Which is the most anticoagulant of choice for

them. Some of these compounds are

(1) lysozyme, a mucolytic polysaccharide that attacks bacteria and causes them to

dissolute.

(2) basic polypeptides, which react with and inactivate certain types of gram-positive

bacteria.

(3) the complement complex that is described later, a system of about 20 proteins that can

be activated in various ways to destroy bacteria; and (4) natural killer lymphocytes that

can recognize and destroy foreign cells, tumor cells, and even some infected cells.

Acquired (Adaptive) Immunity

In addition to its generalized innate immunity, the human body has the ability to

develop extremely powerful specific immunity against individual invading agents

such as lethal bacteria, viruses, toxins, and even foreign tissues from other animals.

This is called acquired or adaptive immunity.

Acquired immunity is caused by a special immune system that forms antibodies

and/or activated lymphocytes that attack and destroy the specific invading organism

or toxin.

Basic Types of Acquired Immunity

Two basic but closely allied types of acquired immunity occur in the body.

In one of these the body develops circulating antibodies, which are globulin

molecules in the blood plasma that are capable of attacking the invading agent. This

type of immunity is called humoral immunity or B-cell immunity (because B

lymphocytes produce the antibodies).

The second type of acquired immunity is achieved through the formation of large

numbers of activated T lymphocytes that are specifically crafted in the lymph nodes

to destroy the foreign agent. This type of immunity is called cell-mediated immunity

or T-cell immunity (because the activated lymphocytes are T lymphocytes).

Gyton 11th edi, Pg440

Q55 The other name for factor XI is:-

(A). Stuart power factor

(B). Calcium

(C).Plasma thromboplastin antecedent

(D).Plasma thromboplastin component

Ans. (C). Plasma thromboplastin antecedent

Repeat from previous AIIMS papers

Page 22: Principles and Practice of Ophthalmology' pg 2875 2013 Q1.All of the ... New Anti-Anginal Drugs Fasudil • Rho kinase inhibitor ... Q7. Which is the most anticoagulant of choice for

Q56 Heat labile instruments for use in surgical procedures can be best sterlized by:- (A).

Absolute alcohol

(B). Ultraviolet rays

(C).Chlorine releasing compounds

(D).Ethylene oxide gas

Ans. (D). Ethylene oxide gas

Greenwood Microbiology 12th edi, pg42

Q57A 60 year old man is diagnosed to be suffering from the legionnaries disease after he

returns home from attending a convention. He could have acquired it.

(A). From a person suffering from the infection while travelling in the aeroplane

(B). From a chronic carrier in the convention center

(C).From inhalation of the aerosols in the air conditioned room

(D).By sharing an infected towel with a fellow delegate at convention.

Ans. (C). From inhalation of the aerosols in the air conditioned room Harrison

15th edi, pg945

Q58. Antibodies are produced by? (A).

Mast cells

(B). Plasma cells (C).T-

cells (D).Langerhans cells

Ans. (B) Plasma cells.

Plasma cell is the antibody secreting cell. It is an oval cell about twice the size of

small lymphocyte.

Eccentrically placed nuclei giving ‗‘CART WHEEL Appearance‘‘

Life span of 2-3 days

A plasma cell makes an antibody of a single specificity, of a single immunoglobulin

class & allotype & of a single light chain only.

Plasma cell develop from activated Blymphocyte

Plasma cell reside in lymphoid organs and mucosal tissues, and some may migrate

to bone marrow and live for many years in this tissue

Q59. Battle's sign is ecchymosis behind the ear involving (a) Fracture of condyle (b) Middle cranial fossa

(c) Anterior cranial fossa (d) Posterior cranial fossa

Ans:B ( Middle cranial fossa)

Battle sign (ecchymosis behind the ear) is associated with a fracture in the petrous portion of the temporal bone, which forms the floor of the middle cranial fossa

Page 23: Principles and Practice of Ophthalmology' pg 2875 2013 Q1.All of the ... New Anti-Anginal Drugs Fasudil • Rho kinase inhibitor ... Q7. Which is the most anticoagulant of choice for

Battle's sign: It is relevant in middle cranial fossa fracture — bruising /ecchymosis over the mastoid process in the line of posterior auricular artery is a tell-tale sign of

underlying middle cranial fossa fracture (temporal bone fracture)

The examiner should check for Battle's sign, an area of ecchymosis behind the ear that

can occur with basilar skull fractures, particularly in the middle cranial fossa, and

temporal bone fractures FONSECA,Oral and maxillofacial trauma - Volume 1 - Page 212

Q60. The superior and inferior ophthalmic veins drain into the

a. Internal jugular vein b. Pterygoid plexus

c. Frontal vein

d. Facial vein

Ans D. Facial vein The superior and inferior -ophthalmic veins drain into the facial vein and

cavernous-:sinus.

The facial vein (or anterior facial vein) is a relative large vein in face. It commences at the side of the root of the nose and is a direct continuation of the angular vein where it

also receives a small nasal branch. It lies behind the facial artery and follows a less

tortuous course. It receives blood from the external palatine vein before it either joins

the anterior branch of the retromandibular vein to form the common facial vein, or

drains directly into the internal jugular vein.

A common misconception states that the facial vein has no valves, but this has been contradicted by recent studies. Its walls are not so flaccid as most superficial veins.

Q61. The masseter originates from the

a. Condyle of the mandible

b. Infratemporal crest of the sphenoid bone

c. Inferior border of the zygomatic arch

d. Pyramidal process of the palatine bone Ans C. Inferior border of the zygomatic arch The masseter originates from the inferior border of the zygomatic arch;

specifically,its superficial head and deep head originate from the anterior two thirds

or posteriorone third of the inferior border, respectively. Its superficial head inserts

into the lateral surface of the angle of the mandible; its deep head inserts into the

ramus andbody of the mandible.

Q62. Which of the following muscles adducts the vocal cords?

a. Lateral cricoarytenoid.

b. Posterior cricoarytenoid. c. Cricothyroid. d. Vocalis.

Ans A. Lateral cricoarytenoid.

The oblique and transverse arytenoids and thyroartenoid also adduct the vocal

folds. The posterior cricoarytenoids abducts the vocal cords. The cricothyroid muscle raises the cricoid cartilage and tenses the vocal cords.

The lateral cricoarytenoid (also anterior cricoarytenoid) muscles extend from the

lateral cricoid cartilage to the ipsilateral arytenoid cartilage. By rotating the arytenoid

cartilages medially, these muscles adduct the vocal cords and thereby close the rima

Page 24: Principles and Practice of Ophthalmology' pg 2875 2013 Q1.All of the ... New Anti-Anginal Drugs Fasudil • Rho kinase inhibitor ... Q7. Which is the most anticoagulant of choice for

glottidis, protecting the airway. (Their action is antagonistic to that of the posterior cricoarytenoid muscles.) The lateral cricoarytenoid muscles receive innervation from the

recurrent laryngeal branch of the vagus nerve (CN X).

Q63. A 55-year old male presents with renal failure. He gj* a history of mild bone pains for the last 7 years. X- ray pelvis

shows osteolytic lesions. Serum electrophoresis reveals a M spike. Peripheral blood evaluation show a rouleaux

formation of RBCs with 35% plasma cells Bone marrow examination showed a increase in plasma cells with aberrant

antigen expression. The most likely diagnosis is:

(a) Monoclonal gammopathy of undetermined significance

(b) Multiple myeloma (c) Smoldering multiple myeloma

(d) Plasma cell leukemia Ans. (2) Multiple myeloma

R e f : H a r r i s o n ' s „ P r i n i c p l e s o f I n t e r n a l M e d i c i n e Chap 111

Monoclonal Gammopathy of Undetermined Significance (MGXJS)

M protein in serum <30 g/L

Bone marrow clonal plasma cells <10%

No evidence of other B cell proliferative disorders

No myeloma-related organ or tissue impairment (no end organ damage, including bone lesions)

Asymptomatic Myeloma (Smoldering Myeloma)

M protein in serum > 30 g/L and/or

Bone marrow clonal plasma cells > 10%

No myeloma-related organ or tissue impairment (no end organ damage, including bone lesions) or

symptoms

Symptomatic Multiple Myeloma

M protein in serum and/or urine

Bonemarrow (clonal) plasma cells or plasmacytoma (If flow cytometry is perf ormed, an(>90%) will

show a "neoplastic"

Q64. True about Bell's palsy is true of the following except?

1. Unilateral facial nerve palsy

2. Herpessimplex virus is commonly implicated

3. Steroids are the treatment of choice 4. Surgical decompression should be performed immediate

Ans. (4) Surgical decompression should be perforate immediately Ref: P: L. Dhingra's 'Diseases of Ear, Nose and Throat5/

Chap 14; Harrison's 'Prinicplesof Internal Medicine'; 18/ Chap 376

Tears flow down from the eye (epiphora). Pain in the ear may precede or accompany the nerve paralysis.

It is defined as idiopathic, unilateral, peripheral facial paralysis or paresis of acute onset. Both sexes are affected with equal frequency.

Any age group may be affected though incidence rises with increasing age.

Page 25: Principles and Practice of Ophthalmology' pg 2875 2013 Q1.All of the ... New Anti-Anginal Drugs Fasudil • Rho kinase inhibitor ... Q7. Which is the most anticoagulant of choice for

« A positive family history is present in 6-8% of patients. Risk of Bell's palsy is more in diabetics (angiopathy) and pregnant women (retention of fluid).

Herpes simplexvirus has also been commonly implicated. Clinical Features

The onset is sudden.

Patient is unable to close his eye. On attempting to close the eye, eyeball turns up and out (Bell's phenomenon).

Saliva dribbles from the angle of mouth. Face becomes asymmetrical.

Q65. Which is not a school dental nurse duty? (a) oral examination

(b) prophylaxis

(c) take x-ray

(d) flouride application

Ans:C(take X-ray )

Q66) In patient with palatal oburator occlusion plan of complete denture should be

(a) balanced on affected side

(b) monoplane

(c) lingualized

(d) cusp in fossa

Ans.B(monoplane)

Q67) Maximum stability while scaling and root treatment is provided by

(a) Scaler

(b) Hoe scaler

(c) Chiael

(d) Files

Ans. Hoe Scaler

Q68) Calculate present in anterior region of mandible is-

(a) Brushite

(b) Magnesium whitlockite

(c) Calcium phosphate

(d) Hydroxyapatite

Ans.d Hydroxyapatite

Q69) Interstitial arthroplasty material for TMJ ANKYLOSIS includes all except-

(a) Osseograft

(b) Buccal fat

(c) Temporalis

(d) Abdominal pad

Page 26: Principles and Practice of Ophthalmology' pg 2875 2013 Q1.All of the ... New Anti-Anginal Drugs Fasudil • Rho kinase inhibitor ... Q7. Which is the most anticoagulant of choice for

Ans:a Osseograft

Q70. Fusion of the premaxilla occurs at:

A. Birth B. & week post natally C. 18 months

D. 32 weeks I.U

Ans D. 32 weeks I.U Ref:- National Board

71. The lateral lingual swellings and tuberculum impar give rise to:-

A. Anterior 1/3rd of tongue B. Anterior 2/3rd of tongue C. Posterior 2/3rd of tongue

D. Lateral side of the tongue

Ans B. Anterior 2/3rd of tongue

Tongue

Because the mandibular arch grows more rapidly than the others, this component of the pharyngeal floor is longer craniocaudally than that of the other arches. By stage 14,

three swellings are apparent on its surface: a small median elevation, the tuberculum impar or median tongue bud, and paired mandibular (lingual) swellings. The

mandibular swellings fuse with each other and with the tuberculum impar to form the

anterior two-thirds of the tongue. A sulcus forms along the ventral and lateral margins

of this elevation and deepens, internal to the future alveolar process of the mandible, to

form the linguogingival groove, while the elevation constitutes the anterior or oral

(presulcal) part of the tongue. Caudal to the tuberculum impar, the copula and the hypobranchial (hypopharyngeal) eminence, form in the pharyngeal floor, and the

ventral ends of the second, third and fourth pharyngeal arches converge onto them. The

third arch component of the hypopharyngeal eminence grows over the second arch and

approaches the anterior tongue rudiment; the two parts fuse along a V-shaped line

corresponding later to the sulcus terminalis. Another transverse groove separates its caudal (fourth arch) part to delineate the epiglottis.

During this sequence of events, the second arch is excluded from the tongue. The

sulcus terminalis has its apex at the foramen caecum: this is a small median diverticulum that forms at the time of fusion of the constituent parts of the tongue, and is the site of downgrowth of the median rudiment of the thyroid gland. Failure of

downgrowth of all or part of the thyroid gland from this site results in ectopic thyroid tissue within the tongue (lingual thyroid), between the foramen caecum and the

epiglottis. The tongue primordia fuse at stage 17 (15 mm), and the sulcus terminalis is formed by the line of fusion. The vallate papillae appear around 10 weeks (52 mm),

and increase in number until the end of the second trimester. Serial reconstructions suggest that the territory of the glossopharyngeal nerve extends considerably beyond

these papillae. However, in general the composite character of the mucous membrane

of the tongue is reflected by its sensory innervation. The anterior, oral part is

innervated by the lingual branch of the mandibular nerve, and by the chorda tympani of the facial nerve. The posterior, pharyngeal part of the tongue is innervated by the

glossopharyngeal, the nerve of the third arch, and the root of the tongue, near the epiglottis, is innervated by the vagus. During stages 14 and 15, the mesenchyme of the tongue is invaded by cells from the occipital myotomes which migrate from the lateral

aspects of the myelencephalon. They pass ventrally round the pharynx to reach its floor accompanied by the hypoglossal nerves. The posterior parts of the tongue and

epiglottis, i.e. the embryonic pharyngeal floor derivatives of the third, fourth and sixth

arches, lie within the oropharynx. Striations appear on the surface of the tongue early in

Page 27: Principles and Practice of Ophthalmology' pg 2875 2013 Q1.All of the ... New Anti-Anginal Drugs Fasudil • Rho kinase inhibitor ... Q7. Which is the most anticoagulant of choice for

the fetal period. The tongue is short and broad, and its entire surface lies within the oral cavity. The posterior third of the tongue descends as the hyoid bone and larynx descend

during the first postnatal year, and by the fourth or fifth year it forms part of the

anterior wall of the oropharynx. Ref:-BDC 3rd edi, pg 213

72. Taste buds are most commonly present in A. Filiform papillae

B. Fungiform papillae

C. Filiform and fungiform papillae

D. Circumvallate papillae and the wall of adjacent trench Ans D. Circumvallate papillae and the wall of adjacent trench

Taste buds

Taste buds are microscopic barrel-shaped epithelial structures which contain chemosensory cells in synaptic contact with the terminals of gustatory nerves. They

are numerous on all types of lingual papillae (except filiform papillae), particularly

on their lateral aspects. Taste buds are not restricted to the papillae, and are

scattered over almost the entire dorsal and lateral surfaces of the tongue and, rarely,

on the epiglottis and lingual aspect of the soft palate. Each Taste buds is linked by synapses at its base to one of three cranial nerves which carry taste, i.e. the facial,

glossopharyngeal or vagus. They share some physiological features with neurones,

for example action potential generation and synaptic transmission, and are therefore

often referred to as paraneurones.

There is considerable individual variation in the distribution of Taste buds in humans. They are most abundant on the posterior parts of the tongue, especially

around the walls of the circumvallate papillae and their surrounding sulci, where there is an average of 250 Taste buds for each of the 8–12 papillae. Over 1000

Taste buds are distributed over the sides of the tongue, particularly over the more

posterior folds of the two foliate papillae, whereas they are rare, and sometimes

even absent, on fungiform papillae (≤3 per papilla). Taste buds have been

described on the fetal epiglottis and soft palate but most disappear from these sites

during postnatal development. Microstructure of Taste buds

Each Taste buds is a barrel-shaped cluster of 50–150 fusiform cells which lies within an oval cavity in the epithelium and converges apically on a gustatory pore,

a 2 μm wide opening on the mucosal surface. The whole structure is about 70 μm in

height by 40 μm across and is separated by a basal lamina from the underlying lamina propria. A small fasciculus of afferent nerve fibres penetrates the basal

lamina and spirals around the sensory cells. Chemical substances dissolved in the

oral saliva diffuse through the gustatory pores of the Taste buds to reach the taste

receptor cell membranes, where they cause membrane depolarization.

Fungiform papillae Fungiform papillae occur mainly on the lingual margin but also irregularly on the

dorsal surface, where they may occasionally be numerous (Fig. 30.11). They differ

from filiform papillae because they are larger, rounded and deep red in colour, this

last reflecting their thin, non-keratinized epithelium and highly vascular connective

tissue core. Each usually bears one or more Taste buds on its apical surface. Foliate

papillae Foliate papillae lie bilaterally in two zones at the sides of the tongue near the sulcus

terminalis, each formed by a series of red, leaf-like mucosal ridges, covered by a

non-keratinized epithelium. They bear numerous Taste buds Circumvallate papillae

Circumvallate papillae are large cylindrical structures, varying in number from 8 to

12, which form a V-shaped row immediately in front of the sulcus terminalis on the

dorsal surface of the tongue. Each papilla, 1–2 mm in diameter, is surrounded by a

Page 28: Principles and Practice of Ophthalmology' pg 2875 2013 Q1.All of the ... New Anti-Anginal Drugs Fasudil • Rho kinase inhibitor ... Q7. Which is the most anticoagulant of choice for

slight circular mucosal elevation (vallum or wall) which is separated from the papilla by a circular sulcus (Figs 30.12, 30.13). The papilla is narrower at its base

than its apex and the entire structure is generally covered with non-keratinized

stratified squamous epithelium. Numerous Taste buds are scattered in both walls

of the sulcus, and small serous glands (of von Ebner) open into the sulcal base. Ref:-Grays Anatomy 39th edi, pg588

73. Ptosis may occur due to damage to:- A. Trochlear nerve

B. Occulomoter nerve

C. Superior oblique muscle

D. Trigeminal nerve

Ans B. Occulomoter nerve Ref:-BDC 4th edi, pg 28

74. The floor of mouth and suprahyoid muscles are supplied by:

A. Lingual B. Inferior alveolar C. Maxillary

D. Facial

Ans A. Lingual

LINGUAL ARTERY

The tongue and the floor of the mouth are suppled by the Lingual Artery which arises from the anterior surface of the external carotid artery. Path

It first runs obliquely upward and medialward to the greater cornu of the hyoid bone. It then curves downward and forward, forming a loop which is crossed by the hypoglossal

nerve, and passing beneath the Digastricus and Stylohyoideus it runs horizontally forward,

beneath the Hyoglossus, and finally, ascending almost perpendicularly to the tongue, turns forward on its lower surface as far as the tip, under the name of the deep lingual artery (profunda linguae ).

It also supplies palatine tonsil.

75. All of the following statements about the vagus are true except that it:

A. Supplies heart and lung B. Carries postganglionic parasympathetic fibers

C. Innervates right two third of transverse colon D. Stimulates peristalsis and relaxes sphincters.

Ans B. Carries postganglionic parasympathetic fibers

76. The tendon of sartorius, gracilis, and semitendinosus muscles forms a Pes Anserinus at

the neck of tibia.

Similar kind of structure is also seen in?

(a) Parotid

(b) Submandibular (c) Cheek

(d) TMJ Ans. (a). Parotid

Facial Nerve

• After entering the parotid gland, the facial nerve branches into a plexus at the pes anserinus,

subdividing thegland into a lateral and median portion.

• This anatomical subdivision provides an important landmarks during surgery of the parotid gland. The facial nerve is identified at its trunk and isolated.

• Surgical removal of the portion of the gland lateral to the pens anserinus is called a lateral

Page 29: Principles and Practice of Ophthalmology' pg 2875 2013 Q1.All of the ... New Anti-Anginal Drugs Fasudil • Rho kinase inhibitor ... Q7. Which is the most anticoagulant of choice for

parotidectomy. • Medial to the pes are branches of the external carotid artery (superficial temporal artery,

transverse facial artery) and venous vessels that drain into the internal jugular vein.

• Lymphatic drainage of the parotid gland is through several intraglandular and periglandular lymph nodes to the submandibular and deep jugular nodal chains. Pes Anserinus and Beyond

• After exiting the stylomastoid foramen, the facial nerve passes anterior to the posterior belly

of the digastric muscle; lateral to the styloid process, external carotid artery , and posterior facial vein; runs anteriorly for about 2 cm before bifurcating into an upper and lower divisions; both divisons run through the substance of the parotid gland, usually passing over the

external jugular vein. • The marginal mandibular and cervical branches may pass deep or split with a twig running

deep and another superficial to the vein. The nerve is approximately 3 mm in diameter between

where it exists from the stylomastoid foramen to where it enters the parotid gland. The facial

nerve is invariabley found at a point where the tip of the mastoid process, cartilaginous

auditory canal, and superior border of the posterior belly of the digastric muscle meet.

77. Food with maximum cholesterol content? (a) Coconut oil,

(b) Ghee,

(c) Hydrogenated fat, (d) Eggs

Ans. (d). Eggs

• Food with maximum cholesterol content is eggs. It is repeat from Medical AIPGMEE papers Eggs and organ meat are rich sources of dietary cholesterol. The cholesterol content of eggs

(500mg/100gm) may be compared with that of liver (440mg/100 gm), kidney (800mg/100gm) and brain (200mg/100 gm)

―The yolks of eggs have the most cholesterol of any food with 1234 mg per 100 gm serving or 411% of the DV. A single egg yolk will provide 210 mg(70% DV) of cholesterol‖- above mentioned

website [DV : daily value; daily value

of cholesterol is 300 mg/day]

Also know : • All the cholesterol in eggs is found in their yolks. Egg white does not have any cholesterol.

• Egg contains all the nutrients except carbohydrate and vitamin C.

• Egg protein is considered the best among food proteins and is the standard against which the quality of other proteins is compared. • NPU of egg protein is 100.

78. Most common single cause of pyrexia of unknown origin:

(a) Mycobacterium tuberculosis

(b) Salmonella paratyphi (c) Brucella

(d) Salmonella typhi Ans. (a). Mycobacterium tuberculosis

(General Problems in Infectious Disease)

Fever of unknown origin :

∗To fulfill the criteria for fever of unknown origin, a patient must have had - An illness of 3 weeks duration

Page 30: Principles and Practice of Ophthalmology' pg 2875 2013 Q1.All of the ... New Anti-Anginal Drugs Fasudil • Rho kinase inhibitor ... Q7. Which is the most anticoagulant of choice for

- Fever over 38.30C on several occasions - And remain undiagnosed after 1 week of study in the hospital.

∗The intervals specified are arbitrary ones intended to exclude patients with protracted but self limited viral illnesses and to allow time for usual radiographic, serologic and cultural studies to be performed.

Etiology

∗Common cause – Most cases represent unusual manifestation of common diseases and not rare or exotic disease e.g., T.B., Endocarditis, Gall bladder disease and H.I.V. are more common cause of pyrexia of unknown origin.

Classification of causes of pyrexia of unknown origin In

adults - Infection (25-40% cases) - Cancers (25-40% cases)

In children – Infections (30-50% cases)

- Cancers (5-10% cases) Infection associated with Pyrexia of unknown origin

∗Both systemic and localized infections can cause pyrexia of unknown origin.

∗Tuberculosis and endocarditis are the most common systemic infections but mycoses, viral disease (particularly infection with Epstein Barr Virus and cytomegalovirus, toxoplasmosis, Brucellosis, Salmonellosis, Malaria, Q. fever, cat scratch disease) have also been implicated.

∗Primary infection with human immunodeficiency virus or opportunistic infections are also associated with Pyrexia of unknown origin.

Neoplasm associated with Pyrexia of unknown origin

∗Many cancers can present as fever of unknown origin

∗ 79. Shy Dragor Syndrome is (a) Vasovagal syncope

(b) Orthostatic hypotension (c) Intestinal polyps

(d) Acute adrenergic shock Ans. (b). Orthostatic hypotension

• Shy-Drager syndrome (SDS) is a rare condition that causes progressive damage to the

autonomic nervous system.

• A neurological syndrome associated with orthostatic hypotension. • The autonomic nervous system controls vital involuntary body functions such as heart rate, breathing, and intestinal, urinary, and sexual functions. The autonomic nervous system also

controls skin and body temperature, and how the body responds to stress. Shy-Drager syndrome leads to dizziness or fainting when standing up, urinary incontinence, impotence,

and muscle tremors.

80. Hypertonic salt solutions are used in all except.

(a) Burn

(b) Brain injury patient (c) Prolonged bowel surgery

(d) Cardiac type IV shock

Ans. (d). Cardiogenic type IV shock

Hypertonic salt solutions are used in prolonged bowel surgery, burns, brain injuries ie. patients

predisposed to tissue edema Hypertonic Salt Solutions

Page 31: Principles and Practice of Ophthalmology' pg 2875 2013 Q1.All of the ... New Anti-Anginal Drugs Fasudil • Rho kinase inhibitor ... Q7. Which is the most anticoagulant of choice for

• Hypertonic salt solutions are less commonly used, and their sodium concentrations range from 250 to 1200 mEq/L. The greater the sodium concentration, the less the total volume is

required for satisfactory resuscitation.

• This difference reflects the movement owing to osmotic forces of water from the intracellular space into the extracellular space. • The reduced volume of water injected may reduce edema formation; this could be crucial in

patients predisposed to tissue edema (e.g., prolonged bowel surgery, burns, brain injuries).

81. According to 2010 ACLS Guidelines all are true except:

(a) Cardiac massage resuscitation immediately after defibrillation without waiting for assessing rhythm

(b) Defibrillation whenever done, it should be done with maximum available energy

(c) Immediately call for help in witnessed or non-witnessed cases (d) Atropine is the drug of choice for asystole

Ans. (d). Atropine is the drugs of choice for asystole

Drugs taken off routine use

• Atropine has been removed from the recommended list of pharmacological agents for the treatment of asystole and pulseless electrical activity. There is lack of evidence for its benefit for

these conditions.Atropine, however, continues to be the drug of first choice for treatment of

haemodynamically significant bradycardia.

• Though short-acting, it has been clearly demonstrated to improve heart rate and symptoms in such patients.It is given in a dose of 0.6 mg intravenously and may be repeated at 3 to 10 minute intervals up to a maximum dose of 2.4 mg.

• In patients requiring cardiac pacing as a result of severe atrio-ventricular block, it can be

used as a temporizing measure to keep heart rates up while preparations are being made for

the pacemaker. • Calcium administration for cardiac arrest is no more recommended regardless of rhythm.

However, in patients with hyperkalemia, especially in the presence of ECG changes, intravenous calcium administration in the form of calcium chloride 5 – 10 ml over 2 - 5

minutes or calcium gluconate 15 - 30 ml over 2 – 5 minutes is recommended

82. Hypotonic fluid is:

(a) Normal saline (b) Ringer lactate

(c) Mannitol (d) 5% dextrose

Ans. (d) 5%Dextrose

IV Fluids • IV fluids are classified as isotonic, hypotonic, or hypertonic solutions, depending on the effect

a fluid has on the ICF and ECF compartments.

Isotonic Solutions

• Isotonic solutions are used to expand ECF volume. Fluid initially stays in the intravascular

compartment. These solutions contain the same concentration of solute to fluid as that in body fluids and exert the same osmotic pressure as ECF in a normal, steady state. Isotonic fluids

are indicated for intravascular dehydration. • Normal saline (or 0.9% NS), lactated Ringer‘s solution, and 5% dextrose in water all function

as isotonic solutions. If an isotonic solution is infused into the intravascular system, fluid

volume increases. One liter of isotonic solution expands the ECF by 1 L. Three liters of isotonic

fluid is required to replace 1 L of blood loss. Hypotonic Solutions

• Hypotonic solutions exert less osmotic pressure than the ECF. Infusion of excessive

Page 32: Principles and Practice of Ophthalmology' pg 2875 2013 Q1.All of the ... New Anti-Anginal Drugs Fasudil • Rho kinase inhibitor ... Q7. Which is the most anticoagulant of choice for

hypotonic fluids can lead to intravascular fluid depletion, hypotension, cellular edema, and cell damage. Because these solutiosn can cause serious complications, the patient and the infusion

should be monitored closely. The hypotonic solution of 0.45% sodium chloride and 0.3% sodium chloride provide free water, sodium and chloride to aid the kidneys in the excretion of solutes. Hypotonic solutions are administered for cellular dehydration.

83. Enamel lamellae is

(a) Uniform arrangement of enamel rods (b) Dentinal tubule in enamel

(c) Enamel projection in dentin

(d) Elevation on the outer surface of enamel

Ans. (c). Enamel projection in dentin

84. Dental lamellae may be predisposing locations for caries because (a) They are rich in organic material and form a road of entry for bacteria

(b) They are rich in inorganic material and form a road of entry for bacteria (c) They act as semi permeable membrane

(d) None of the above

Ans. (a). They are rich in organic material and form a road of entry for bacteria

ENAMEL TUFTS AND LAMELLAE

• Enamel tufts and lamellae may be likened to geologic faults and have no known clinical significance. They are best seen in transverse sections of enamel.

• Enamel tufts project from the dentinoenamel junction for a short distance into the enamel.

They appear to be

branched and contain greater concentrations of enamel proteins than the rest of the enamel. • As a special protein called tuft protein has been reported at these sites, tufts are believed to

occur developmentally because of abrupt changes in the direction of groups of rods that arise from

different regions of

the scalloped dentinoenamel junction.

• Lamellae extend for varying depths from the surface of enamel and consist of linear, longitudinally oriented defects filled with organic material. This organic material may derive from trapped

enamel

organ components or connective tissue surrounding the developing tooth.

• Tufts and lamellae are usually best demonstrated in ground sections, but they also can be

seen in carefully demineralized sections of human enamel because of their higher protein content. Cracks in the enamel sometimes can be mistaken for lamellae but can be distinguished from the latter because they generally do not contain organic material. Ref:- Tencate‟s Oral Histology 8th edition, page 157

85 . Water content in enamel by volume is:- (a) 2 to 3 %

(b) 6 to 7% (c) 10 to 12%

(d) 20 to 25 %

Ans. (b). 6 to 7%

86. Water content in enamel by weight is:-

(a) 2 to 3 %

(b) 6 to 7%

Page 33: Principles and Practice of Ophthalmology' pg 2875 2013 Q1.All of the ... New Anti-Anginal Drugs Fasudil • Rho kinase inhibitor ... Q7. Which is the most anticoagulant of choice for

(c) 10 to 12% (d) 20 to 25 %

Ans. (a). 2 to 3%

Enamel is translucent, and varies in color from light yellow to gray-white. It also varies in

thickness, from a maximum of approximately 2.5 mm over working surfaces to a feather edge

at the cervical line.

– This variation influences the color of enamel because the underlying yellow dentin is seen

through the thinner regions. – Fully formed enamel consists of approximately 96% mineral and 4% organic material and water .

– The inorganic content of enamel is a crystalline calcium phosphate (hydroxyapatite)

substituted with

carbonate ions, which also is found in bone, calcified cartilage, dentin, and cementum. – Various ions—strontium, magnesium, lead, and fluoride—if present during enamel formation, may be incorporated into the crystals.

– The susceptibility of these crystals to dissolution by acid provides the chemical basis for

dental caries. – The high mineral content renders enamel extremely hard; this is a property that together

with its complex structural organization enables enamel to withstand the mechanical forces

applied during tooth functioning. – This hardness also makes enamel brittle; therefore an underlying layer of more resilient dentin is necessary to maintain its integrity . If this supportive layer of dentin is destroyed by

caries or improper cavity preparation, the unsupported enamel fractures easily.

Q87 . True about Tome’s fibers (a) Cytoplasmic extensions in dentinal tubules, (b) Periodontal fiber extending into root dentine.

(c) Cytoplasmic process of odontoblast extending into the enamel

(d) Periodontal fibers ending into root cementum Ans. (a). Cytoplasmic extensions in dentinal tubules,

• Coincident with this deposition of collagen, the plasma membrane of odontoblasts adjacent to

the differentiating ameloblasts extends stubby processes into the forming extracellular matrix. • On occasion one of these processes may penetrate the basal lamina and interpose itself

between the cells of the inner enamel epithelium to form what later becomes an enamel

spindle. • As the odontoblast forms these processes, it also buds off a number of small, membrane-

bound vesicles known as matrix vesicles, which come to lie superficially near the basal lamina.

The odontoblast then develops a cell process, the odontoblast process or Tomes‘ fiber, which is left behind in the forming dentin matrix as theodontoblast moves away toward the pulp). • The mineral phase first appears within the matrix vesicles as single crystals believed to be

seeded by phospholipids present in the vesicle membrane.

• These crystals grow rapidly and rupture from the confines of the vesicle to spread as a cluster of crystallites that fuse with adjacent clusters to form a continuous layer of mineralized matrix.

The deposition of mineral lags behind the formation of the organic matrix so that a layer of organic matrix, called predentin, always is found between the odontoblasts and the

mineralization front. • Following mineral seeding, noncollagenous matrix proteins produced by odontoblasts come

into play to regulate mineral deposition. In this way coronal mantle dentin is formed in a layer approximately 15 to 20 _m thick onto which then is added the primary (circumpulpal) dentin. Ref:- Tencate‟s Oral Histology 8th edition, page 172

• Tomes process can be defined as that part of the ameloblast apical(or distal) to the apical

terminal bars.It contains numerous secretion granules and is usually devoid of endoplasmic reticulum and mitochondria.Tomes process can be divided into two portions a proximal and

Page 34: Principles and Practice of Ophthalmology' pg 2875 2013 Q1.All of the ... New Anti-Anginal Drugs Fasudil • Rho kinase inhibitor ... Q7. Which is the most anticoagulant of choice for

distal part. • The proximal part of tomes process contacts adjacent ameloblasts. The distal part ,also called

the interdigitating part is surrounded by (or interdigitates with enamel).Acquisition of tomes

process signals the beginning of the secretory stage of amelogenesis. Oral Development and Histology, James Every”Third edition Page 89”

• Tomes Processes. The Tomes process is a small pyramidal cytoplasmic extension at the

distal end of each ameloblast, and it is marked off from Stratum Intermedium “James Handerson, Introduction to Dental Anatomy - Page 177”

Q.88. According to scientists recent studies says about pattern of enamel prism demineralization, true is:-

(a) Head and tail both are resistant to dissolution

(b) Head and tail both are not resistant to dissolution

(c) Head is more resistant to dissolution (d) Tail is more resistant to dissolution

Ans. (d). Tail is more resistant to dissolution

Although the dissolution process occurs more in the head regions of the rod, the tail regions

and the periphery of the head regions are relatively resistant to acid attack.

• Human enamel is composed of rods or prisms that in transverse section are shaped with a

rounded head or body section and a tail section. • Structural components of enamel prism are millions of small, elongated apatite crystals. The

long axis of the apatite crystallites within the central region of the head (body( is aligned almost parallel to the rod long axis,

and the crystallites incline with increasing angles (650) to the prism axis in the tail region.

• The susceptibility of these crystallites to acid section to be correlated with their orientation.

Although thedissolution process occurs most in the head regions of the rod, the tail regions and the periphery of the head regions are relatively resistant to acid attack. Ref : Sturdevant's Art and Science of Operative Dentistry By Theodore Roberson, Harald

O. Heymann, Edward J.

Q.89 . In Ricketts esthetic plane lower lip rests:

(a) Rest on plane (b) 1mm posterior to plane

(c) 2mm posterior to plane (d) 1mm anterior to plane

Ans. (c) 2mm posterior to the plane

In rickets plane lower lips rests 2mm posterior to the plane and upper lip rests 4mm to the plane

Ricketts‟ E line - aesthetic Plane:- • A quick method to look at one profile is to imagine a line tangent from the lower chin to the

nosetip.Cephalometrically, Ricketts‘ E line is drawn from the tip ofthe nose to soft tissue chin.

Normal values suggest that the upper lip is 4 mm behind the E line while the lower lip lies 2

mm behind this referenceline. It is important to mention that this reference line is influenced a

great deal by the growth of the noseand also varies with age and sex.Ricketts recommended

that lip position should be analysed with the nosechinreference. These values are for Caucasians and it is obvious that cannot be applied to all races.

Page 35: Principles and Practice of Ophthalmology' pg 2875 2013 Q1.All of the ... New Anti-Anginal Drugs Fasudil • Rho kinase inhibitor ... Q7. Which is the most anticoagulant of choice for

Q90. In a child diagnosis of tongue thrusting is made by? (a) Holding paper in between lips

(b) Lower lip is held lightly by thumb & finger and ask to swallow water

(c) Holding a piece of paper in front of nose (d) Examine the digits of patients

Ans. (b). Lower lip is held lightly by thumb & finger and ask to swallow water

In a child diagnosis of tongue thrusting can be made by making lower held lightly by thumb

and finger and ask to swallow water

Q91. Ester L.A is contraindicated in: (a) Cholinesterase deficiency

(b) Coagulation disorders

(c) Diabetic neuropathy/ nephropathy (d) Hyperparathyroidism

Ans. (a). Cholinestrase deficiency

• In cases of a relative contraindication it is preferable to avoid administration of the drug in question because of an increased risk that an adverse response will develop. An alternative

drug that is not contraindicated is recommended. However, if an acceptable alternative is not

available, the drug in question may be used, but judiciously, with use of the minimum dose

that will provide adequate pain control. • One example of a relative contraindication is the presence of atypical plasma

(pseudo)cholinesterase, which decreases the rate of biotransformation of ester local anesthetics. The amides may be used with no increase in risk in these patient. “Handbook of Local Anesthesia - Page 72-STANLEY MALAMED”

Q92. If impacted 3rd molar is to be extracted in patient planed for bilateral saggital split osteotomy then extraction should be done

(a) At the time of surgery

(b) 1 month after surgery

(c) 6 month before surgery (d) 8-12 weeks after surgery when complete healing occurred

Ans. (c). 6 Month before the surgery

―Removal of mandibular third molars 6 months prior to the osteotomy allows time for the sockets to heal, which decreases the chance of a bad split.‖

• Ideally third molar should be removed 6 months to 1 year prior to BSSO, when this is not possible the osteotomy for the ascending ramus should be modified.If removed less than 6

months prior to orthognathic surgery,it causes incomplete stage of bone healing and will cause unfavourable splitting of mandible.

“Oral and maxillofacial surgery, Raymond J fonseca Volume II Page 305”

Q93. A 30year old male in road traffic accident is in shock, immediate IV fluid for

resuscitation:? (a) Joule's solution (b) Ringer lactate solution

(c) LMW dextran

(d) Normal saline

Ans. (b). Ringer Lactate • Usually after trauma, hypovolemic shock is developed due to severe blood loss (around 40%

loss). Ringer lactate solution because of its high osmotic value maintains the fluid in vascular compartment.

• Principles of advanced trauma life support (ABCD) should be maintained in the initial assessment in theemergency department (A-airway maintainance; B- breathing or

maintainance of respiraration; Ccirculation;

Page 36: Principles and Practice of Ophthalmology' pg 2875 2013 Q1.All of the ... New Anti-Anginal Drugs Fasudil • Rho kinase inhibitor ... Q7. Which is the most anticoagulant of choice for

D—drug) • The first and most critical obligation is to make the airway patent.

• The tongue,which may have a tendency to fall back,must be controlled, and objects obstructing the airway must be removed.

• If an obstruction cannot be removed, a new airway must be established by endotracheal

intubation. After the airway has been secured and respiration is occurring, vital signs must be assessed, including pulse rate and blood pressure.

• Any significant blood loss is likely to be coming from injuries apart from those of the face.

Other critical injuries must be ruled out, including intracranial hemorrhages, cervical and other spinal injuries. Lactated Ringer‘s solution (Hartmann‘s solution) contains all the following electrolytes chloride, sodium, potassium in approximately the same proportion as human

extracellular fluid except bicarbonate (MAHE 95). • Fluid replacement in hypovolemic shock therefore starts with a widely available balanced

electro-lyte solution that can be stored right where it needed, is at room temperature, and can

be given to anybody.Ringer‘s lactate is by far the most common solution used. Crystalloid solutions (e.g., Ringer‘s lactate solution) are ideal in situations where water and sodium loss is predominant and serve as initial treatment in haemorrhagic shock

Ref: V.Kapoor‟s Textbook of oral & maxillofacial surgery 2nd ed, p 608)

Q94. The laryngeal mask airway is used for screening the airway of patient in all of following except? (a) In a difficult intubation

(b) In cadriopulmonary resusication

(c) In child undergoing an elective eye surgery

(d) In child with large tumour in oral cavity Ans. (d). In children with large tumor in oral cavity

Yildiz et al concluded it that Mallampati class 4, male patients, history of snoring, increasing age, and increasing weightwere found to be risk factors for DMV in his study. Langeron et al

suggested five recognized criteria as independent factors for a DMV. The Five Predictors of

difficult bag and mask ventilation and Oxygenation, can be

summarized in the word ―OBESE‖. The Five Predictors of difficult bag and mask ventilation and Oxygenation, can be summarized in the word ―OBESE‖

1. The Obese (body mass index > 26 kg/m2)

2. The Bearded (Option B)

3. The Elderly (older than 55 y) (Option D)

4. The Snorers (Option C) 5. The Edentulous

(Age older than 55 yr, body mass index > 26 kg/m2, beard, lack of teeth, and history of

snoring), the presence of two indicating high likelihood of DMV (sensitivity, 0.72; specificity, 0.73)... A simple DMV risk score was established.

Being able to more accurately predict DMV may improve the safety of airway management.

Murphy M, Walls RM. Identification of the difficult and failed airway. In: Walls RM, Murphy MF, Luten R, eds. Manual of emergency airway management. Philadelphia: Lippincott, Williams, Wilkins; 2004:70-81.

Q95) A 60 years old patient having cardio respiratory arrest in hospital ward, for oxygenation

what should be

done immediately? .

(a) Nasal intubation

(b) Tracheostomy

Page 37: Principles and Practice of Ophthalmology' pg 2875 2013 Q1.All of the ... New Anti-Anginal Drugs Fasudil • Rho kinase inhibitor ... Q7. Which is the most anticoagulant of choice for

(c) Cricothyroid membrane puncture (d) Oral intubation

Ans. (d). Oral intubation

• Performing tracheostomy in a 'hospital ward' that too in a serious patient who requires O2 immediately is neither advised nor feasible

Q96)During prolonged oral surgical operation which of the following is used to secure

patent airway? (a) Cuffed nasotracheal tube

(b) Non-cuffed nasotracheal tube

(c) Proseal LMA (d) LMA

Ans. (a). Cuffed nasotracheal tube

• Types of endotracheal tube include oral or nasal, cuffed or uncuffed, preformed (e.g. RAE (Ring, Adair, and Elwyn) tube), reinforced tubes, and double-lumen endobronchial tubes.

• For human use, tubes range in size from 2 to 10.5 mm in internal diameter (ID). The size is

chosen based on the patient's body size, with the smaller sizes being used for pediatric and neonatal patients. Tubes larger than 6 mm ID usually have an inflatable cuff.

• Originally made from red rubber, most modern tubes are made from polyvinyl chloride.

• Those placed in a laser field may be flexometallic. Robertshaw (and others) developed double-

lumen endobronchial tubes for Thoracic surgery. These allow single-lung ventilation while the other lung is collapsed to make surgery easier.

Q97. Split lingual bar type connector is not preferred in contemporary partial denture, major disadvantage is

(a) Complex design (b) Entrapment of tissues

(c) Decreased retention

(d) Inadequate stability

Ans. (a). Complex design LINGUAL BAR WITH CONTINUOUS BAR INDIRECT RETAINER SYNONYMS: KENNEDY BAR, SPLIT LINGUAL BAR, DOUBLE LINGUAL BAR

Indications : Situations where the major connector must contact the natural teeth to provide bracing and

indirect retention and there are open cervical embrassures which contraindicate the use of a lingual plate. There

must be adequate space

for the lingual bar portion of the major connector. Contraindications :

1. Where a lingual bar or lingual plate will suffice.

2. Any contraindication for a lingual bar.

3. Any contraindication for a lingual plate except open cervical embrassures. 4. Diastemas.

Advantages :

1. More rigid than lingual bar.

2. Covers less tooth and tissue surface than lingual plate.

Disadvantages : 1. Very complex design.

2. May be objectionable to patient because there are four edges exposed to the tip of the tongue.

Page 38: Principles and Practice of Ophthalmology' pg 2875 2013 Q1.All of the ... New Anti-Anginal Drugs Fasudil • Rho kinase inhibitor ... Q7. Which is the most anticoagulant of choice for

Ref:- Clinical Removable Partial Prosthodontics - Page 240 Points to

Remember:-

LABIAL BAR (OR PLATE) Indcations:

1. Lingually inclined teeth preventing the use of a lingual mandibular major connector. 2. Lingual tori or exostoses which can not be removed surgically, avoided in the RPD design, or

covered by the

framework with adequate relief. 3. A lingual major connector can not be used because of the slope or undercut of the lingual

alveolus. 4. The patient can not tolerate a lingual major connector.

5. Diastemas and open cervical embrasures contraindicating a lingual plate.

Contraindications: 1. A lingual major connector may be used.

2. Facial tori or exostoses. 3. The facial alveolar ridge is undercut.

4. High facial muscle attachments which would result in less than 3 mm of space between the superior edge of the

labial bar and the marginal gingiva of the teeth.

Advantages:

1. Can be used where lingual major connector can not

Disadvantages: 1. A labial major connector is longer than a lingual major connector and, therefore, must be

wider and/or thicker or larger to provide the necessary rigidity.

2. A labial major connector may be visible when the patient smiles and it may distort lip contour resulting in poor esthetics.

3. Difficult to add prosthetic teeth to framework.

CINGULUM BAR

Indications: 1. Height of activated lingual frenum and floor of the mouth at the same level as marginal gingiva.

2. Inoperable tori or exostoses at the same level as the marginal gingiva.

3. Severely undercut lingual alveolus

4. Concern that a major connector traversing the gingival sulcus will cause a periodontal

problem. 5. Considerable gingival recession. Contraindications:

1. When a simpler major connector may be used.

2. Diastemas and open cervical embrasures where the metal will show.

Advantages:

1. Can be used where lingual bar and lingual plate can not.

2. Does not traverse the marginal gingiva or overlay the lingual alveolus. 3. Easy to add prosthetic teeth to framework. Disadvantages:

1. Must be bulky to have sufficient rigidity and thus may be objectionable to the patient.

Q.98. Which of the following is not a type of major maxillary metal partial denture frame work:- (a) Lingual U plate

(b) Palatal strap (c) Palatal plate

(d) Single palatal bar

Page 39: Principles and Practice of Ophthalmology' pg 2875 2013 Q1.All of the ... New Anti-Anginal Drugs Fasudil • Rho kinase inhibitor ... Q7. Which is the most anticoagulant of choice for

Ans. (a). Lingual U plate

Maxillary Major Connectors

Six basic types of maxillary major connectors are considered:

1. Single palatal strap (Option B)

2. Combination anterior and posterior palatal strap–type connector 3. Palatal plate-type connector (Option C)

Q.99 . Property of internal occlusal rest are all except

(a) Horizontal stabilization

(b) Retention (c) Vertical stop

(d) Support

Ans. (b). Retention

Q.100. Occlusal rest seat in the RPD are prepared primarily to

(a) Protect the occlusal surfaces (b) Stabliize the RPD

(c) Resist lateral Chewing forces (d) Resist vertical forces of occlusion

Ans. (d). Resist vertical forces of occlusion

Q.101. Advantage of internal occlusal rest are following, except?

(a) Provide retention

(b) Eliminates visible clasp arm

(c) Rest seat is located in a more favorable position (d) Provides support and stabilization Ans (a) Provide retention

Internal Occlusal Rests

• A partial denture that is totally tooth supported by means of cast retainers on all abutment

teeth may use

intracoronal rests for both occlusal support and horizontal stabilization.

• An intracoronal rest is not a retainer and should not be confused with an attachment.

• Occlusal support is derived from the floor of the rest seat. • Horizontal stabilization is derived from the near-vertical walls of this type of rest seat. • The form of the rest should be parallel to the path of placement, slightly tapered occlusally,

and slightly

dovetailed to prevent dislodgment proximally.

• The main advantages of the internal rest are that it facilitates the elimination of a visible clasp arm buccally

and permits the location of the rest seat in a more favorable position in relation to the tipping axis (horizontal)

of the abutment.

4. U-shaped palatal connector

5. Single palatal bar (Option D) 6. Anterior-posterior palatal bars

Mandibular Major Connectors

The six types of mandibular major connectors include the following: 1. Lingual bar

2. Linguoplate 3. Sublingual bar

4. Lingual bar with cingulum bar (continuous bar)

5. Cingulum bar (continuous bar)

6. Labial bar

Page 40: Principles and Practice of Ophthalmology' pg 2875 2013 Q1.All of the ... New Anti-Anginal Drugs Fasudil • Rho kinase inhibitor ... Q7. Which is the most anticoagulant of choice for

Ref:- Mccracken Prosthodonti

Q102) Which of the following structure is not present at floor of third ventricle ?

(a) Optic stalk

(b) Third nerve

(c) Infundibulum

(d) Mamillary body

Ans. (b) Third nerve Ref.

BDC 5th/Vol-III/428

Q103Boundaries of facial recess are formed by all of the following except :

(a) Facial nerve

(b) Stapedius tendon

(c) Chorda tympani

(d) Short process of incus

Ans. (b) Stapedius tendon

Ref. Dhingra 5th/6

Q104. Elevation at the urethral crest in prostatic urethra is elevation formed by :

(a) Prostatic glands

(b) Elongation of detrusor

(c) Trigone muscle attachment

(d) Preprostatic internal sphincter

Ans. (a) Prostatic glands

Ref. BDC 5th/Vo.-II/379

105) . A fertilised egg should be implanted in/Embryo is implanted in uterus by?

(a) 14 days,

(b) 28 days.

(c) 8 weeks

(d) 24 weeks

Ans:A(14 Days)

A fertilized egg should be implanted in by 14 days.

106. For vertical method of obturation, technique employed is?

(a) Soften by heat and vertically condense to fill the lateral canal in all three dimensions

(b) Warm the GP point over flame and vertically condense it using chemicals

(c) Warm the GP point and condense using sharp pointed spreaders

(d) Soften by chemicals and condense vertically by warm heated pluggers

Page 41: Principles and Practice of Ophthalmology' pg 2875 2013 Q1.All of the ... New Anti-Anginal Drugs Fasudil • Rho kinase inhibitor ... Q7. Which is the most anticoagulant of choice for

Ans:A( Soften by heat and vertically condense to fill the lateral canal in all three

dimension).

Warm vertical condensation

Schilder introduced warm vertical compaction as a method of filling the radicular

space in three dimensions. Preparation requirements for the technique include preparing a canal with a continuously tapering funnel and keeping the apical

foramen as small as possible.

The armamentarium includes a variety of pluggers and a heat source.

Schilder pluggers come in a variety of sizes (#8 =0.4 mm, # 8 1/2 = 0.5 mm, etc.,

for sizes #9, # 9 1/2 , #10, #10 1/2,#11, #11, 1/2, #12) with increasing diameter. The instruments are marked vertically at 5-mm intervals.

The technique involves fitting a master cone short of the corrected working length

(0.5 to 2 mm) with resistance to displacement .This ensures that the cone diameter is larger than the prepared canal. Nonstandard cones that closely replicate the

canal taper are best because they permit the development of hydraulic pressure during compaction. After the adaptation of the master cone it is removed and

sealer is applied. The cone is placed in the canal and the coronal portion is

removed with heat. A heated spreader or plugger is used to remove portions of the

coronal gutta-percha and soften the remaining material in the canal.

Advantages of warm vertical compaction include filling of canal irregularities and accessory canals.

Disadvantages include a slight risk of vertical root fracture because of compaction forces, less length control than with lateral compaction, and the potential for extrusion of material into the periradicular tissues. Warm vertical compaction is

difficult in curved canals,where the rigid pluggers are unable to penetrate to the necessary depth. To allow the rigid carriers to penetrate within 4 to 5 mm of the

apex, the canals must be enlarged and tapered more, in comparison with the lateral compaction technique;however, excessive removal of tooth structure weakens the root.

Warm vertical compaction of gutta-percha employs heat and various condensers.

A, Nonstandard cones are selected and fit short of the prepared length because they more closely replicate the prepared canal. B, Heated pluggers or spreaders are used to apply heat to the master cone and remove the excess coronal material. C, A

room temperature plugger is used to compact the heated gutta-percha. D, Apical

compaction is complete. E, A gutta-percha segment is placed in the canal, and

heat is applied. F, The heated segment is compacted. G, The process is repeated for

the coronal portion of the canal by placing and heating a segment of gutta-percha.

H, A plugger is again used to compact

the heated material. I, Completed obturation

Vertical condensation or warm gutta percha technique:-

Introduced by schilder

Used with step back technique of root canal preparation

After placing gutta percha vertical force is applied with heated pluggers so that the gutta pecha cones fill the lumen of canal in all the three dimension upto apical

foramen

Risk of vertical root fracture is main disadvantage of this technique

107) Hepatitis E is associated with?

(a) Chronic liver failure

(b) Cirrhosis

(c) Acute liver failure

Page 42: Principles and Practice of Ophthalmology' pg 2875 2013 Q1.All of the ... New Anti-Anginal Drugs Fasudil • Rho kinase inhibitor ... Q7. Which is the most anticoagulant of choice for

(d) Acute renal failure

Ans. (c) Acute liver Failure

Hepatitis E is associated with chronic liver disease.

― Hepatitis E ocassionally develops into acute, severe liver disease, and is fatal in

about 2 % of the cases. Clinically it is comparable to hepatitis A, but in pregnant

woman the disease is often severe and is associated with clinical syndrome called

fulminant hepatitis failure. ― Hepa titis E is u sual l y se lf limiti ng bu t ma y dev el op i nt o f ulmi n ant he pati ti s‖ (

Acute liver failure)

Hepatitis E virus:- Also known as entericallly transmitted non A-nonB(NANB)virus or epidemic NANB

It is classified in the genus Herpes virus under the family calciviridae

It is A RNA virus with single stranded positive Sense RNA

It is transmitted by feco-oral route

In india heaptits E is responsible for majority of epidemic and sporadic hepatitis in

adults

An epidemiological feature that distinguish HEV from other enteric agents is rarity

of secondary person to person transmission (Secondary attack is very low 2-3 % as

against 10-20% in HAV virus

A unique feature is the clinical severity and high case fatality rate of 20-40% in

pregant woman especially in the last trimester of pregnancy

It is charactersticallly associated with cholestasis

Hepatitis C is the commonest cause of post transfusion hepatitis and chronic

hepatitis

Most common cause of chronic carrier state:- HCV>HDV>HBV

108) In a patient of HIV ELISA test is done and confirmed by western blot test.

(a) To reduce false positive results (b) Is a highly sensitive test for HIV

(c) To decrease the experimental error during the test

(d) Measure the amount of actively replicating HIV

[Ans. (a) Ref. 7* edition, Ananthnarayan Pfc. 5*\-5W\

ELISA is simple and relatively inexpensive but false positive reactions are not uncommon,

particularly with sera containing Rheumatoid factor, antilymphocyte or other antibodies.

• Sera stored for long periods contain nonspecific ‗sticky‘ immunoglobulins.

• False positive results can also occur in hepatic disease.

• ELISA is a screening test and is ideal for screening several samples at a time.

• It is inconvenient for testing single samples quickly.

• A positive result in any one screening test mav not be accepted without confirmation.

• It was the practice to use for confirmation the western blot test which was considered the gold

standard.

As ELISA test can result in false positive results so to remove them it is confirmed by western blot test.

109. Bacteria that grow optimally between 25-40°C are called.

Page 43: Principles and Practice of Ophthalmology' pg 2875 2013 Q1.All of the ... New Anti-Anginal Drugs Fasudil • Rho kinase inhibitor ... Q7. Which is the most anticoagulant of choice for

a. Mesophiles b.Psychrophiles c.Thermophiles

DCryophifcs [Ans. (a) Ref. Ananthnarayan, 7* edition, Pg. Mesophilic:

Bacteria which grow best at temperatures of 25-4<fC are called mesophILIC

In this group some bacteria like pseudomonas aeruginosa have a wider range (5-43°C).

110. Coefficient of thermal expansion of metal used for PFM crowns should be ?

A. Same as porcelain B. More than porcelain

C. Less than porcelain D. More or equal to porcelain

[Ans. (D) Ref. Phillip's 10* Ed Pg 602,603]

The alloys and porcelains used for the construction of such restoration have a number of stnngent

requirements For example, both the metal and the ceramic have coefficients of thermal contraction that are closely matched such that the metal has a slightly higher value, if undesirable residual tensile stresses

in porcelain are to be avoided (Please note that material with higher coefficient of thermal

contraction also have higher coefficient of thermal expansion and vice a versa)

Q111 Botulism toxin causes ?

A. Tachycardia B. Bilateral cranial nerve involvement

C. Fever D. Hypertension

[Ans. (B) Ref. Ananthanarayan 7thEd Pg 263]

Toxin- It acts by blocking the production or release of acetylcholine at the synapses and neuromuscular

junctions CAUSING dysarthria due to cranial nerve involvement

Q112)A cell membrane is damaged by insertion of a microneedle, repair shall occur by which of the following

processes?

(a) Hydrophobic interaction

(b) Lateral movement of proteins (c) Enzyme-catalyzed process

(d) An active process involving hydrolysis of ATP

Ans. (2) Lateral movement of proteins Ref: McNeil PL, Steinhardt RA. Loss, restoration, and maintenance of plasma membrane integrity. J Cell Biol. 1997;137(l):l-4; http://ivurw.ncbi.nlm.nih.gov/pmc/articles/ PMC2139822/

• Living, nucleated cells respond to microneedle punctures of their plasma membranes by rapidly (within sec) resealing the breach created.

• The mechanism used is Ca2+ dependent and hypothesized to be an active process governed by specific protein-protein interactions that result in a local exocytotic reaction. As a result of this exocytotic response, new membrane is added locally to the site of cell surface

injury

Q113)Which of the following injections is available for subcutaneous administration?

(a) Albuterol lipf 1 (b) Terbutaline

(c) Metaproterenol

(d) Pirbuterol

Page 44: Principles and Practice of Ophthalmology' pg 2875 2013 Q1.All of the ... New Anti-Anginal Drugs Fasudil • Rho kinase inhibitor ... Q7. Which is the most anticoagulant of choice for

Ans: (2) Terbutaline I Ref: http://www.mayoclinic.com/health/drug-informatm DR603123M3g H I

• Terbutaline subcutaneous injection (Brand name Brethine) is used to prevent bronchospasm in patients 12 year of age and older with asthma, bronchitis, emphysema, &other lung diseases

Q114) Vasopressin antagonists act on which part of the nephron?

(a) Proximal convoluted tubule

(b) Distal convoluted tubule

(c) Cortical collecting tubule

(d) Medullary collecting duct Ans. (4) Medullary collecting duct

Ref: Uawithya P, Pisitkun T, Ruttenberg BE, Knepper MA. Transcriptional profiling of native inner medullary collecting duct cells from rat kidney. Physiol Genomics. 2008■ 32(2)- 229-253.

• Vasopressin acts on the inner medullary collecting duct (IMCD) in the kidney to regulate water and urea transport.

• Among the 30 different aquaporin transcripts with the greatest signals on the arrays were 3 water channels: aquaporin-2, aquaporin-3, and aquaporin-4, all of which have been reported to be targets for regulation by vasopressin.

• The V2 vasopressin receptor was strongly expressed, but the Via and Vlb vasopressin receptors did not produce signals above background, indicating that V2 is the predominant receptor in the kidney. Ref: Sands JM, NonoguchiH, Knepper MA. Vasopressin effects on urea and H20 transport in inner medullary coUectmg due subsegments.Am ]Physiol. 1987 Nov; 253:F823-32.

Q115) Insulin resistance is seen in liver disease because of:

(a) Damaged hepatocytes

(b) Decreased secretion of insulin (c) Steatosis

(d) Low C-peptide levels

Ans. (1) Damaged hepatocytes Ref: Robbins & Cotran's 'Pathologic Basis of Disease’; 8/e, Chap 19; http://jcem.endojournals.Org/content/87/7/3019.full

• Insulin resistance is defined as the failure of target tissues to respond normally to insulin. It leads to decreased uptake of glucose in muscle, reduced glycolysis and fatty acid oxidation in the liver, and an inability to suppress hepatic gluconeogenesis.

Q116) Enamel is permeable to?

(a) Bacteria

(b) Bacterial products (c) Peroxides

(d) Retrovirus Ans.(c) Peroxides

The enamel-denlin-pulp environment (Fig. 8-10) represents a uniqlre symbiosis of

mineralized tissues and cells. The enamel of the tooth is virtually all inorganic material (96 wt%) arranged in a crystalline array called enamel rods. Although the enamel

is permeable to some substances, such as the peroxides in bleaching agents, it is

generally not permeable to material components, bacteria, or bacterial products

Page 45: Principles and Practice of Ophthalmology' pg 2875 2013 Q1.All of the ... New Anti-Anginal Drugs Fasudil • Rho kinase inhibitor ... Q7. Which is the most anticoagulant of choice for

―Phillips The science of Dental Material,11th Edition‖Page 181‖

Q117) Response which is graded by an observer on an agree or disagree continuum is based on:

A Visual analog scale B Guttman scale

C Likert scale D. Adjectival scale

Ans C Likert scale

Q118)In fpd patient, while cementing a crown?

A. It is easy to remove because compressive strength of cement is less, shear strength is more

B. it is easy to remove because compressive strength of cement is more, tensile strength is less C. Cement is the weakest part in prosthesis

D. A thin layer of cement has higher failure rates than thicker or has lower success rates

Ans.(a) A. It is easy to remove because compressive strength of cement is less, shear

strength is more

It has been shown that generally greater tensile or shear forces are required to unseat

appliances cemented with luting agents that have a high compressive strength than with cements of low compressive strength

“Skinner's science of dental materials - Page 487,9th ed.”

Q119)Sinus tract exit on buccal or lingual side depends upon? A. Location of root surface to cortical plates B. Thickness of hard tissue

C. Thickness of soft tissue

D. Root dentine

E. Proximity of root to soft tissue

Ans.(a) Location of root surface to cortical plates

Sinus Tracts :-When periapical inflammation is present around the roots of the lower

first and second molars, a sinus tract occurs, mainly on the buccal side. The tract

breaks through on the buccal surface more readily because the roots are closer to the

buccal cortex, despite thedespite the fact that resorption of both cancellous and cortical bone takes place. Around a mandibular third molar, a sinus tract occasionally forms

on the lingual side because of the closer approximation of the roots to the lingual

cortical plate.

―Endodontology 2 Ed: Biologic considerations in endodontic .Procedure, Seltzer‖

Q120.Inca bone, Gothe ossicle in skull presents an independent bone in suture is

A. Pterion

B. Lambda

Page 46: Principles and Practice of Ophthalmology' pg 2875 2013 Q1.All of the ... New Anti-Anginal Drugs Fasudil • Rho kinase inhibitor ... Q7. Which is the most anticoagulant of choice for

C. Bregma D. Asterion

Ans (b) Lambda

An isolated bone at the lambda is named the Inca bone.

Q121) Which dye is most suitable for fungal demonstration in biopsy

a) Alizarin red

b) Veirhoff dye

c) Masson‘s trichrome

d) P.A.S.

Correct answer : d) P.A.S.

Q122) Most common cause of Down’s syndrome is?

A. Translocation

B. Mosaicism

C. Paternal nondisjunction

D. Maternal nondisjunction

Correct answer : D. Maternal nondisjunction

Maternal nondisjunction in meiosis I is the most common cause of Down‘s syndrome (95% of

total cases)

Q123) Which stain is used to study fungal morphology in tissue sections?

A. Periodic acid–Schiff B. Alizarin Red

C. Masson‘s Trichrome

D. Von Kossa

Correct answer : A. Periodic acid–Schiff

Stains used to demonstrate fungi are Periodic Acid Schiff (PAS) and Methenamine Silver Stain

PAS stains both living and dead fungi, whereas Methenamine Silver stains only living fungi

Massons‘s Trichrome is used to demonstrate collage, muscle and fibrin Alizarin Red and Von Kossa are used to demonstrate calcium in tissues

Q124) Which of the following renal calculi is seen associated with proteus infection?

A. Calcium Oxalate

B. Triple Phosphate

C. Xanthine

D. Uric Acid

Correct answer : B. Triple Phosphate

Page 47: Principles and Practice of Ophthalmology' pg 2875 2013 Q1.All of the ... New Anti-Anginal Drugs Fasudil • Rho kinase inhibitor ... Q7. Which is the most anticoagulant of choice for

High ammonia concentration leads to the formation of triple phosphate stones. This arises

when urine is infected with urea splitting organisms like Proteus.

Q125) Rashtriya Swasthya Bima Yojna is:

(a) Government run insurance scheme for its employees

(b) Government run insurance scheme for all citizens

(c) Government run insurance scheme for all poor

(d) Private insurance company run scheme for all poor

I Ans is ‗c‘ i.e. Government run insurance scheme for all poor [Ref: http://www.rsby.gov.in/about_rsby.aspx]

I • Rashtriya Swasthya Bima Yojana (RSBY) is a Government sponsored scheme for the BPL population of India.

• 75% by Central Government

• 25% by State Governments

(Government of India‘s contribution is 90 percent in case of North-eastern states and Jammu and Kashmir.)

• RSBY scheme is under the Ministry of Labour and Employment

• The objective of RSBY is to provide protection to BPL households from financial liabilities arising out of health shocks that involve hospitalization.

‘ RSBY provides for cashless insurance of Rs. 30,000 per family per yearfor hospitalisation in public as

well private hospitals.

1 Coverage extends to five members of the family whidi includes the head of household,

spouse and up to three dependents. Pre-existing conditions are covered from day one and there is no age fimit. '

Q126)A Study revealed lesser incidence of carcinoma colon in pore vegetarians than non

vegetarians by which it was concluded that p-carotene is protective against cancer. This may

not be true because the vegetarian subjects may be consuming high fibre diet which is

protective against cancer. This is an example of:

(a) Multifactorial causation

(b) Causal association (c) Confounding factor

(d) Common association

Ans is ‘c* i.e. Confounding factor [Ref:Park21/ep68,84]

In an epidemiological study, in addition to the exposures that the study is investigating, there

may be other factors that is associated with the exposure and independently affects the risk of developing disease. If the prevalence of these other factors differ between groups being

compared, they will distort the observed association between the disease and exposure under study. These distorting factors are called confounding factors or variables.

In the study given, protective factor is being studied. But as high fibre content of vegeterian food is protective against CA colon, it can confound or distort the observed protective role of p-carotene present in

vegetarian food. Thus this is an example of confounding factor.

Page 48: Principles and Practice of Ophthalmology' pg 2875 2013 Q1.All of the ... New Anti-Anginal Drugs Fasudil • Rho kinase inhibitor ... Q7. Which is the most anticoagulant of choice for

Q127) Which of the following is best suited for the role of social worker ?

(a) Health professional involved in physiotherapy

(b) Health professional involved in coping strategies,Linterpersonal skills, djnstment with family

(c) A person involved in finding jobs and economic support far disabled

(d) Health professional

Ans is ‗b‘ Le. Health professional involved in coping strategy interpersonal skills, adjustment with family [Ref:Park21/ep642j

• The medical social worker is a paramedical worker who has bn trained in social case

work, and in tfie art of interviewing people.

• The medical social worker forms a link between the institution where is employed and the

community.

• The principal work of the medical social worker is to visit the and probe into the personal, economic and social causes of illness*** collect social history of the patient with a view to supplemeat medical history.

• The medical social worker also tries to secure help to the patient through the community resources. In chronic disabling illnesses like tuberculosis, leprosy and the medical social worker helps in the rehabilitation of the patient

Q128) Which of following is the best for treatment of communicated fracture ?

a) 2.4mm reconstruction plates with central hole

b) Dynamic screws / dynamic compression plates with eccentric screws

c) Multiple 2.00mm monocorticalplates

d) Multiple 1.5mm monocortical plates Ans A. 2.4mm reconstruction plates with central hole

Atleast three types of fracture fixation possible for transverse symphyseal plate. A dental splint including at least three teeth on either side of fracture

incombination with four hole miniplate ( 2.0)placed just underneath the apices

of root apex. Two four hole miniplates ( 2)fixed with monocortically applied screws for

superior plate and bicortical screws for inferior plate.The superior plate

neutralize the tension zone

Two or three long lag screws that cross the fracture in a horizontal direction.These scres must be generally be longer than 20mm.For an optimum placement of the screw the countersink must be given.Both screwsare placed in

bone below the tooth roots Stronger plates such as reconstruction plates may be necessary when these

fractures occurs in combination with subcondylar fracture or pan facial fracture.

Based on champy ideal osteosynthesis lines at symphyseal region two plates are required.

An ideal osteosynthesis lines for mandibular body corrensponds top course of line of

tension at base of alveolar process.In thisregion aplate can be fixed with monocortical self tapping screws.

Behind the mental foramen , a platecan be applied immediately below the dental roots ad above the inferior alveolar nerve

Page 49: Principles and Practice of Ophthalmology' pg 2875 2013 Q1.All of the ... New Anti-Anginal Drugs Fasudil • Rho kinase inhibitor ... Q7. Which is the most anticoagulant of choice for

At the angle of jaw the plate is most favorably placed on broad surface of external oblique line as high as possible.

In the anterior region between the mental foramen in addition to cervical plate,

another plate near the lower border of mandible is necessary in order to neutralize torsional forces.Second plate is applied parallel to first plate with a gap of 4.5mm between them

“M an u al of i nt er n al fi x at i on i n cr ani of aci al s ke l et o n by J.pr ei n 1 99 8”

“Te xt boo k of or al a nd m axi l l of a ci al s ur g er y by N el l i m a M al i k”

Q129.A condylar fracture with fragment overlap of >5mm and angle >37 degree needs what

treatment? a) Closed reduction and IMF ( Intermaxillary fixation) b) ORIF ( Oral reduction and intraoral fixation)

c) Soft diet only

d) No treatment

Ans B. ORIF ( Oral reduction and intraoral fixation)

Greatly displaced and dislocated fractures separation of fragments exceeding 5mm and

displacement exceeding 30 degree) increasingly are being treated surgically, a functional

conservative treatment is worth considering in less severly dislocated fractures.

“Pet er W ar d boo t h , Ma xi l l of aci al s ur g er y”

Q130. Which of the following are most complicated fractures of mandible is? a. Symphysis

b) Body

c) Condyle d) Angle

Ans D. Angle

Unfavorable lines of fracture from treatment vpoint of view in the angle region

favours superior & medial displacement of the posterior fragment. Grossly displaced mandibular angle fracture produced by open reduction can be

fixed by eccentric dynamic compressive plating Miniature plates using mono-cortical screws through intraoral or Extraoral

approach

In case of angle fracture there is simultaneous muscle pulling action that adds one

more complication than any other fractured part of mandible. Kruger 6th edi, pg 401-402

Q131. The most common anatomic site of fracture of the mandible is? a) Body

b) Angle

c) Condyle d) Symphysis menti e) Coronoid process

Ans B. Angle

―NBD E PA R T 2 J UL Y 1 98 1, ORAL SURGERY AND PAIN CONTROL-Q4 7‖

Page 50: Principles and Practice of Ophthalmology' pg 2875 2013 Q1.All of the ... New Anti-Anginal Drugs Fasudil • Rho kinase inhibitor ... Q7. Which is the most anticoagulant of choice for

In case of condylar fracture the fracture line runs inside the capsule of temporomandibular joint.These fractures cannot be fixed with plates or

screws.subcodylar fractures are situated below the capsules.As a rule condylar fracture

are treated conservatively,one of reason being danger of necroses of proximal fragment (

condylar head)This is due to interruption of vascular supply denudation during open

reduction procedure “M an u al of i nt er n al r e du ct i on i n cr a ni of a ci al skel t on by J pr ei n P a g e 93 ”

Communited mandibular fractures with bilateral condylar fractures produces crossbite

and tends to increas4s the interangular distances,making accurate reduction challenging.Failure to recognize and correct this increas4d interangular

distance,making accurate reduction challenging. “PETE R WA RD BOO TH MAXI L LOFA CI AL T RAU MA A ND E ST HETI C FA I LU RE A ND

RECO NS TR UC TI VE S UR GERY Pa g e 29 6”

Q132. The weakest part and most vulnerable part of fracture in the mandible is:-(KCET- 2011) A. Ramus of mandible

B. Condylar neck

C. Angle of the mandible D. Region of the canine tooth

Ans B. Condylar neck

Condylar neck is vulnerable to fracture as it is very slender and is actually a safety mechanism which prevents the injury to middle cranial fossa by fracturing itself at the

neck.

Q133.In fracture of atrophic mandible treatment which is best treatment modality ?

a) Bone grafting & load bearing

b) Bone grafting & load sharing

c) Open reduction

d) Semi rigid

Ans A. Bone grafting & load bearing

―As mandible is severely atrophic,so load bearing is better as compared to load sharing‖

In addition to adequate reduction and stabilization of fractured segments the successful

management of fractures involving the edentulous mandible requires that consideration

be given to amount of bone present

When the mandible is severely atrophic, it is possible that healing will not occur even if

ORIF principles properly applied .In some circumstances,treatment consists of

simultaneously bone graft reconstruction at time of fracture repair

As the mandible is severely atrophic so load bearing is better as compared to load

sharing

PETER WARD BOOTH MAXILLOFACIAL TRAUMA AND ESTHETIC FACIAL AND

RECONSTRUCTIVE SURGERY

Load-bearing fixation is a device that is of sufficient strength and rigidity that it

can bear the entire load applied to the mandible during functional activities. Injuries that require load-bearing fixation are comminuted fractures of the mandible, those fractures where there is very little bony interface because of

Page 51: Principles and Practice of Ophthalmology' pg 2875 2013 Q1.All of the ... New Anti-Anginal Drugs Fasudil • Rho kinase inhibitor ... Q7. Which is the most anticoagulant of choice for

atrophy, or those injuries that have resulted in a loss of a portion of the mandible (defect fractures).

In such cases the fixation device must bridge the area of comminution, minimal

bone contact, or bone loss, and bear all of the forces transmitted across the

injured area that are generated by the masticatory system.

Loadbearing fixation is sometimes called bridging fixation because it bridges

areas of comminution or bone loss.

The most commonly used load-bearing device is a mandibular reconstruction

bone plate. Such plates are relatively large, thick, and stiff. They use screws that

are generally greater than 2.0 mm in diameter (most commonly 2.3 mm, 2.4 mm,

or 2.7 mm).

When secured to the fragments on each side of the injured area by a minimum

of three bone screws, reconstruction bone plates can provide temporary stability

to the bone fragments.

The bone plates are not prosthetic devices and will usually fail in time (several

months to years later) by either loosening of the screws or fracture of the plate, but can provide stability until the comminuted fragments have consolidated and/or the missing bone has been replaced with grafts

Load-sharing fixation is any form of internal fixation that is of insufficient

stability

to bear all of the functional loads applied across the fracture by the masticatory

system.

Such a fixation device(s) requires solid bony fragments on each side of the

fracture that can bear some of the functional loads.

Fractures that can be stabilized adequately with load-sharing fixation devices

are simple linear fractures, and constitute the majority of mandibular Fractures.

Fixation devices that are considered load-sharing include the variety of 2.0 mm

miniplating systems that are available from a number of manufacturers

Lag screw techniques are also load-sharing in that the bone that is compressed is sharing the functional loads with the screws.

Page 52: Principles and Practice of Ophthalmology' pg 2875 2013 Q1.All of the ... New Anti-Anginal Drugs Fasudil • Rho kinase inhibitor ... Q7. Which is the most anticoagulant of choice for

Q134.Alveolar bone grafting in a cleft palate patient is done? a) After maxillary expansion, cross bite correction, before canine eruption

b) Before maxillary expansion cross bite correction after canine eruption

c) After maxillary expansion cross bite correction after canine eruption

d) Before maxillary expansion cross bite correction before canine eruption

Ans A. After maxillary expansion, cross bite correction, before canine eruption

As the permanent teeth come in, there is a strong tendency for the maxillary incisors to

erupt rotated and often in crossbite.

The major goal of orthodontic treatment at this time is to correct incisor position, and prepare the patient for an alveolar bone graft.

Although alveolar bone grafts in infancy appear to be contraindicated, placing a bone

graft in the alveolar cleft area before the permanent lateral incisors (if present) or

permanent canines erupt, is advantageous.

This stabilizes the cleft area and creates a healthy environment for the permanent teeth., the permanent lateral so canines should erupt through the graft, which means that the best time to place such a graft is between 7 and l0 years.

Any necessary alignment of incisors or expansion of posterior segments should be

completed before the alveolar grafting. The alveolar graft now is a routine part of contemporary treatment, and doing it at the

right time is critically important. Profitt 4th edi, Pg 323

Bone Grafting of Alveolar Cleft Defects Primary bone grafting refers to bone-grafting procedures involving alveolar cleft defects

in children younger than 2 years of age; this term implies nothing about technique.

Early secondary bone grafting refers to bonegrafting procedures performed between 2 and 4 years of age.

Secondary bone grafting is done between 4 and 15 years of age, and late secondary bone grafting refers to reconstruction of residual alveolar cleft defects in the adult.

Secondary Alveolar Cleft Bone Graft

A successful alveolar cleft bone graft satisfies several objectives.

Page 53: Principles and Practice of Ophthalmology' pg 2875 2013 Q1.All of the ... New Anti-Anginal Drugs Fasudil • Rho kinase inhibitor ... Q7. Which is the most anticoagulant of choice for

In addition to giving bony support for the teeth adjacent to the cleft and providing bone through which teeth can erupt, it offers maxillary arch continuity and aids in closure of

the oronasal fistula. It also supports the alar base of the nose.

Conceptually the technique is not difficult, but technically it can be tedious.

The soft tissue in and adjacent to the cleft side is incised and elevated so that labial and palatal mucosal leaflets are everted to obtain labial and palatal closure. The tissues

lining the cleft are elevated and inverted into the nose for nasal floor closure. Particulate

marrow and cancellous bone harvested from the iliac crest is placed into the cleft defect, filling it from the piriform rim to the alveolar crest before closure of the labial tissues .

Providing bony support to teeth adjacent to the cleft site is of paramount

importance . In most cases, bone should be grafted into the cleft before orthodontic tooth alignment is

begun. When the cleft is filled with normal, viable bone, the orthodontist can proceed

with tooth alignment without fear of exposing a root surface into the cleft site. In fact, after a 2-month healing period, a tooth can be moved

into the newly grafted bone with the expectation that the bone will respond to the tooth movement as any normal bone would. Any tooth movement undertaken before the graft is placed could jeopardize the bony support of the teeth adjacent to the cleft.

As a related consideration, grafted bone that obliterates the alveolar cleft also provides

bone through which teeth can erupt.

When canines and, in some cases, central incisors are allowed to erupt before bone grafting, they often lack adequate periodontal bone support. When the bone graft precedes permanent tooth eruption, compromised periodontal situations can often be

prevented. El Deeb and colleagues studied the eruption patterns of canines through grafted bone

in alveolar cleft defects.They found that canine teeth erupt spontaneously through the

grafted bone but that this eruption may be later than normal and that it takes longer.

However, the point is that canines can and do erupt through the grafts. If eruption seems greatly delayed, surgical and orthodontic intervention is appropriate.

The graft be placed between 9 and 12 years of age when the canine root is one quarter

to half formed.

In the bilateral cleft case, the premaxilla is stabilized as the bone grafts are

incorporated between the premaxilla and the lateral maxillary segments. In the process, the alveolar ridge contour is restored so that the ability to provide a stable, esthetic

prosthesis is enhanced. There is often some degree of collapse of the maxillary arch

form. It is possible to expand the arch after grafting, however, it is preferable to expand

these collapsed segments to as optimal an arch form as possible before grafting. Pregraft expansion also widens the cleft site, which allows better access for nasal floor

closure. After the arch expansion has occurred, the bone graft can be placed. After the graft has been incorporated, it can be expected to maintain a good arch form.

Mcdonald 9th edi, pg 624

Q135.Most common impaction of mandibular third molar is

a) Mesio angular

b) Vertical

c) Distoangular

d) Horizontal

Ans A. Mesio angular

Page 54: Principles and Practice of Ophthalmology' pg 2875 2013 Q1.All of the ... New Anti-Anginal Drugs Fasudil • Rho kinase inhibitor ... Q7. Which is the most anticoagulant of choice for

The impaction generally acknowledged as the least difficult impaction to remove is the mesioangular, particularly when only partially impacted. The mesioangular impacted

tooth is tilted toward the second molar in a mesial direction. This type o[ impaction is the most

commonly seen, making up approximately 43% of all impacted teeth.

In the vertical impaction the long axis of the impacted tooth runs parallel to the long axis of the second molar. This impaction occurs with the second greatest frequency, accounting [or

approximately 38% of all impactions, and is third in difficulty of removal.

Concerning angulation, the three types of maxillary third molars are ( 1 ) the vertical impaction 2) the distoangular impaction (3) the mesioangular impaction (Fig. 9-39, C) . The

vertical impaction occurs approximately 63% of the time, the dis to angular approximately 25%, and the mesioangular position approximately 1 2% of the time. Rarely, other positions, such as a transverse, inverted, or horizontal position, are encountered;

these unusual positions account for less than 1% of impacted maxillary third molars .

Vertical and distoangular impactions are the less complex to remove, whereas mesioangular

impactions are the most difficult ( exactly the opposite of impacted

mandibular third molars).

Mesioangular impactions are more difficult to remove because the bone that overlies the impaction and requires removal or expansion is on the posterior aspect of the tooth, and is

much thicker than in the vertical or distoangular impaction. In addition, access to the mesioangularly positioned tooth is more difficult if an erupted second molar is in place

The mesioangular mandibular impaction is usually the least difficult to remove of the four

basic angulation types. Neelima Malik 2nd edi

Q136.Nitrous oxide is common anaesthesia now a days. nitrous oxide N2O acts by ? a) Non specific CNS depression

b) Gasserian ganglion blockade c) Blocking peripheral pain pathways

d) Block neural trunk

Ans A. Non specific CNS depression

Page 55: Principles and Practice of Ophthalmology' pg 2875 2013 Q1.All of the ... New Anti-Anginal Drugs Fasudil • Rho kinase inhibitor ... Q7. Which is the most anticoagulant of choice for

Q137. Nitrous Oxide that is used as anesthetic agent has mechanism of action as? A. Gasserian ganglion block

B. Block peripheral nerves

C. Reversible (indirect) depression of CNS D. Block neural trunk

Ans C. Reversible (indirect) depression of CNS

―Nitrous oxide produces nonspecific central nervous system (CNS) depression. “

Although it is classed with inhalational general anesthetics, it produces limited analgesia, and

thus surgical anesthesia is unlikely unless concentrations producing anoxia are reached.

Nitrous oxide is the weakest of all inhalation agents, with a minimum alveolar concentration of 104. The minimum alveolar concentration of an inhalation agent is a measure of its potency. It

is the concentration required to produce immobility in 50% of patients. At concentrations between 30% and 50%, nitrous oxide will produce a relaxed, somnolent patient who may

appear dissociated and easily susceptible to suggestion

Nitrous Oxide Color coding for cylinder: Blue

Prepared first by joseph priestly in 1774

It is 1.5 times heavier than the air The boiling point of N2O, which is 88.5C (127 F), indicates that it is a gas at room temperature When compressed into a cylinder, N2O becomes a liquid.

Minimum alveolar concentration 104% and is maximum

Nitrous oxide can leads in megaloblastic anemia due to B12 deficiency. It can leads to bone marrow depression

Also called as Laughing gas( Named by Humphrey Davey)

MAC= 104% It is used in concentration of 50-55% with 33% oxygen

Blood gas coefficient=0.47( Fast induction and recovery)

Diffusion hypoxia occurs when supply of N20 is stopped while recovery from anesthesia

Entonox is Mixture of 50% N20 and 50% O2

It is very good analgesic but weak anesthethic agent

It is colourless, non irritable and non inflammable

N20 Is contraindicated in pneumothorax and volvulus

Methaemoglobinemia and laryngospasm may occur due to impurities like nitric oxide and nitrogen dioxide

It is used as supplement to anesthesia ( It is not a complete anesthesia)

When given with other agents it increases alveolar concentration of that agent ( Second gas effect) and its own concentration ( Concentration effect)

Contraindicated in:-

Pneumothorax,pneumoperitoneum,

Middle ear surgery Postreior fossa surgeries Laryngoscopic surgeries

Eye surgeries

Page 56: Principles and Practice of Ophthalmology' pg 2875 2013 Q1.All of the ... New Anti-Anginal Drugs Fasudil • Rho kinase inhibitor ... Q7. Which is the most anticoagulant of choice for

Points to Remember:-( Visual Analog Scale- Pain Ratings)

Q138. When a minimal injury as a glancing blow is struck to what variable its related ?

a) Angulation of strike

b) Position of strike

c) Location of strike d) Area of strike

Ans A. Angulation of strike

Page 57: Principles and Practice of Ophthalmology' pg 2875 2013 Q1.All of the ... New Anti-Anginal Drugs Fasudil • Rho kinase inhibitor ... Q7. Which is the most anticoagulant of choice for

Glancing means striking obliquely and bouncing off an angle.Glancing blow are combat events that can occur when fighting a mob of equal or higher level

Q139.In lingual splaying of guardman fracture with ORIF, which is clinical feature?

a) Intercanthal distance

b) Interangular distance

c) Go-gn distance d) Interpupillary distance

Ans B. Interangular distance

The type of mandibular fracture more prone for complication is symphyseal fracture associated with condylar fractures and poor dentition. In this type of fracture muscle pull from tongue and suprahyoids can cause lateral flaring of mandibular angle and lingual tipping of the buccal

segments. The buccal fracture line at symphysis remains intact but the linual cortex is separated

Increased flaring is seen when bilateral subcondylar fracture are also present. ―Guardsman fracture is also termed as PARADE FRACTURE‖

“Or al an d M axi l l of a ci a l t r aum a , Raym o u nd J . Fons ec a”

Q140.It is essential to lubricate dam before applying it which is not a suitable lubricant?

a) Soapy water

b) Vaseline c) Shaving cream

d) Scrub gel

Ans B. Vaseline

A water-soluble lubricant applied in the area of the punched holes facilitates the passing of the dam septa through the proximal contacts. A rubber dam lubricant is commercially available,

but other lubricants, such as shaving cream or soap slurry, are also satisfactory. Applying the lubricant to both sides of the dam in the area of the punched holes aids in passing the dam through the contacts. Cocoa butter or petroleum jelly may be applied at the corners of the

patient's mouth to prevent irritation. These two materials, however, are not satisfactory rubber dam lubricants because both are oil based and not easily rinsed from the dam once the dam

is placed.

Lubricant used before applying rubber dam

are:

Soap slurry Shaving cream Scrub gel

Important Notes:- Rubber dam : ( By SC BARNUM-1860)

Parts Feature

Rubber Dam Sheet Rubber Dam sheet of 5*5 or 6*6 inches size.

5types=

Page 58: Principles and Practice of Ophthalmology' pg 2875 2013 Q1.All of the ... New Anti-Anginal Drugs Fasudil • Rho kinase inhibitor ... Q7. Which is the most anticoagulant of choice for

Thin (0.06inches)-0.15mm

Medium= (.08 inches)-0.20mm

Heavy=.010 inches-0.25mm

Extraheavy=.012 inches-0.30mm

Special duty=.014 inches-0.35mm The dull side of sheet faces towards

occlusal side to avoid light

reflection

Rubber Dam Holder Mostly Young holder is used Rubber Dam Clamp 2 Types: winged and wingless

Smaller size is for premolars

Larger size is for molars No212 Clamp is used for retracting

rubber dam sheet during class 5

restorations, its bow is placed

toward distal surface

Rubber dam punch

Clamp holding device

The rubber dam is particulary adapted to primary 2nd molar because cervical constriction favour its retention

The distance between 2 holes in a rubber dam sheet is ideally 6.3mm

Common rubber dam used for pediatric use:-

Partially erupted permanent molar:154A,8Aivory

Fully erupted permanent molar- 14,8Ivory

Suggested retainers for various Anchor tooth applications:-

Retainer Application

W56 Most molar anchor teeth W7 Mandibular molar anchor teeth W8 Maxillary molar anchor teeth W4 Most premolar anchor teeth W2 Small premolar anchor teeth

W27 Terminal mandibular molar anchor

teeth requiring preparations involving distal surface

Sturdvent 4th edition

Q141.Which of following is wrong method to apply rubber dam with wingless clamp on the

tooth? a) Place clamp on tooth and insert the dam over it

b) Place dam on tooth with fingers then place clamp over it c) Place dam & frame outside the oral cavity & then on tooth using forcep OVER the dam

d) Place dam & frame outside the oral cavity & then on tooth using forcep UNDER the dam

Page 59: Principles and Practice of Ophthalmology' pg 2875 2013 Q1.All of the ... New Anti-Anginal Drugs Fasudil • Rho kinase inhibitor ... Q7. Which is the most anticoagulant of choice for

Ans C. Place dam & frame outside the oral cavity & then on tooth using forcep OVER the dam

Vimal Sikri 1st edition Page 102

The rubber dam placement can follow different patterns like

First place winglessclampon the tooth.Stertch the lips of appropriate hole in rubber dam sheet and then slide it over bow and jaws of clamp and around cervical of the tooth

Place wingless clamp and rubber dam together around the tooth.For this he rubber dam is passesd over the bow of the clamp.The clamp forces enagegsthe holes in the

jaws beneath the dam and is used to place the clamp along with the dam over the tooth .The dam is then stretched and slide over the jaws onto the tooth

USE WINGED CLAMP.Both the clamp and dam can be placed together.For this rubber

dam is placed over the bow.The clamp forceps enagegs the holes in the jaw and is used to place the clamp along with the rubber dam over the tooth.The dsam is then stretched and slide onto the cervical areas of the tooth

First place the rubber dam on the tooth and then secure it in position by placing a winged or wingless clamp

Q142.Material of choice for restoring non-carious erosive lesion of the cervical areas of the tooth?

(a) GIC (b) RM-GIC

(c) Compomer

(d) Composite

Ans (d) Composite

DIRECT PICK FROM E- QUESTIONS

Q143.Material of choice in Class V cavity with abfraction?

(a) GIC

(b) RM-GIC

(c) Compomer

(d) Microfilled Composite

Ans (d) Microfilled Composite

Q144. Most stable restoration for Class Vunstable erosive lesion?

(a) GIC

(b) RM-GIC

(c) Compomer

(d) Composite Ans (b) RM-GIC

“Sturdvent 4Th edition Page 478,Repeat from AIIMS May 2012 paper

Q145. Fluoride released from GIC restoration is replaced by ? a) Hydoxyl ion

b) Aluminium ion c) Silicate ion

d) Carboxylate ion

Page 60: Principles and Practice of Ophthalmology' pg 2875 2013 Q1.All of the ... New Anti-Anginal Drugs Fasudil • Rho kinase inhibitor ... Q7. Which is the most anticoagulant of choice for

Ans A. Hydoxyl ion

It is direct pick from E course Mcq, University of Birmingham, Question No.9 Section , GIC

Q146. Mechanism of adhesion of GIC restoration over tooth surface is by means of? a)Carboxyl group

b)Chelates with Metal ions

c)C=C double bond d)Polymer Chains Ans:a ( CARBOXYL GROUP)

The mechanism by which the glass ionomer bond to tooth structure has not been clearly identified.However there seems little doubt that it primarily involves chelation of carboxylic

groups of the polyacids with calcium in apatite of enamel and dentin.The adhesive

menchanism of glass ionomer is comparable to that of zinc polycarboxylate cement.The bond

strength to enamel is always higher than that to dentin because of greater inorganic content of

enamel and its greater homogenicity from morphological standpoint Phillips 11th edition page 475”

The same carboxylic acid side group also are capable of chelating surface ions on glass

particles or calcium ions from the tooth.This process generates true chemical bonds at internal and external interface in the reaction conditions are correct

Sturdvent 4th edition page 207

Q147. In caries,which structure becomes more prominent ?

a) Striae of retzius b) Lines of pickerill

c) Hunter shreger lines

d) Stria of wickhem

Ans A. Striae of retzius

“Indeed, the striae appear to be accentuated in early lesions.”

Enamel rods are formed linearly by successive apposition of enamel in discrete increments. The resulting variations in structure and mineralization are called the incremental striae of Retzius

and can be considered growth rings .In horizontal sections of a tooth, the striae of Retzius

appear as concentric circles. In vertical sections, the lines transverse the cuspal and incisal areas in a symmetric arc pattern descending obliquely to the cervical region and terminating at

the DEJ. When these circles are incomplete at the enamel surface, a series of alternating grooves, called the imbricationlines of Pickerill, are formed. The elevations between the grooves

are called perikymata; these are continuous around a tooth and usually lie parallel to the

cementoenamel junction and each other.

Caries preferentially attacks the cores of the rods and the more permeable striae of Retzius, which promotes lateral spreading and undermining of the adjacent enamel. Indeed, the striae

appear to be accentuated in early lesions.

Points to Remember:-

Sturdvent , The art and science of operative dentistry, 4th edition.

Page 61: Principles and Practice of Ophthalmology' pg 2875 2013 Q1.All of the ... New Anti-Anginal Drugs Fasudil • Rho kinase inhibitor ... Q7. Which is the most anticoagulant of choice for

1mm square of cut dentin exposes 30,000-45,000 dentinal tubules Carbide bur are more cool cutting than steel bur

If pulp temperature increases by 11F –It may cause destructructive reaction in pulp

Most deterious speed is 3,000-30,000 rpm

Instrument pressure should not be more than 4 oz when using high speed and

12 oz with low speed.

Khn of Dentin ( 68) Enamel=(343)ie. Enamel is 5 times harder than dentin. Run out- Is the eccentricity of maximum displacement of bur head from its axis

of rotation. Maximum eccentricity is .023mm Speed Frequency

Conventional/Low speed <10,000rpm

High Speed 10,000-15,000 rpm Ultra Speed >1,50,000rpm

The time for progression from incipient caries to clinical caries (cavitation) on smooth surfaces is estimated to be 18 months ± 6 months

Histopathology of caries:The first change is loss of interprismatic or inter-rod substance of enamel

The earliest change is accentuation of incremental stria of retzius due to loss of minerals and prominence of organic structures

Zones of enamel caries

Translucent zone (TZ)

First recognizable zone of alteration

Advancing front of the lesion

Half the lesions demonstrate TZ not always present

Seen in long GS in clearing (quinolone-RI-1.62)

TZ appears structureless

Pore volume -1% (compared to 0.2% of sound enamel

Dark zone

Lies adjacent and superficial to translucent zone

Positive zone

Shows positive bifringence (in contrast to sound enamel)

Pore volume of 2-4% (polized light)

Presence of small pores:large molecules of quinolone are unable to penetrate

Micropore system-gets filled with air and become dark

Medium like water may penetrate

Body of lesion

Between unaffected surface and dark zone

Area of greatest demineralization

Pore volume-5% in periphery and 25% in centre

Quinolone imbibitions-body appears transparent

Water imbibitions-+birefringence compared to sound enamel

Stria of retzius- prominent

Surface zone

Page 62: Principles and Practice of Ophthalmology' pg 2875 2013 Q1.All of the ... New Anti-Anginal Drugs Fasudil • Rho kinase inhibitor ... Q7. Which is the most anticoagulant of choice for

Quantitative studiesopartial demineralization of 1-10%

Pore volume-less than 5% of the spaces

Negative biferingence-water imbibitions

Positive biferengence-porous subsurface

Histological zone of enamel caries

Q148.After amalgam polishing the outermost surface has crystal arranged flattened to tooth surface This is known as?

a) Beilby layer

b) Weelby layer

c) Sealby layer d) Teilby layer

Ans A. Beilby layer

After amalgam polishing, the outermost surface has crystals arranged flattened to tooth

surface. This layer is known as a) Beilby layer

The polished l ayer is t her ef or e t h ou ght of as one ma de up min ut e

cr yst als, t hu s kn own as t hemi cr o- cr yst all i ne layer of Beil by lay er (name d af t er a s cient ist who f ir st not e ds u ch a s ur f ace l ayer af t er

p olishing whi ch is e a sily k ept cl ean a nd h as a high c o r r o s i o n

r e s i s t a n c e )

Points to remember about cavity preparation for Amalgam

No.245 bur with heaf length of 3mm and diameter of 0.8mm is used to prepare the class I cavity

Ideal depth of occlusal cavity is 1.5mm for amalgam

An occlusal cavosurface bevel is contraindicated in cavity prreparation of amalgam restoration

Spherical type of amalgam is recommended in teeth having direct pulp

exposure because less condensation pressure is needed With high copper amalgam posts carve burnishing is not indicated

because burnishing technique has been shown to have no significant

effect on clinical performanece of high copper amalgam

The total axial depth of gingival floor should be minimum of 0.8mm

Gingival margin should clear adjacent tooth by 0.5mm

Force applied for condensation amalgam is 20-3kg/1.4-1.8 pounds Speed in polishing causes overheating ( above 60C)may damage pulp and

leads to evaporation of mercury Discoloration of malagm is due to silver sulphide film as results of

chemical corrosion and high copper malagam copper oxide

Minimum mercury technique/Eames techniqwue – greater plasticity ad well adapted to cavity walls

After trituration and condensation of amalgam it undergoes contraction

Page 63: Principles and Practice of Ophthalmology' pg 2875 2013 Q1.All of the ... New Anti-Anginal Drugs Fasudil • Rho kinase inhibitor ... Q7. Which is the most anticoagulant of choice for

followed by expansion Delayed expansion occurs due to moisture contamination during

trituration and condensation and occurs 3-4 days due to accumulation

of hydrogen gas 400 micrometres Condensation is done to bring condenstaed unattached gamma phase as

closely as possible .Force required 2-3 kg

Indium is present in fifth generation amalgam alloys

Residual mercury in set malgam is 45-50%

In cavity preparation for amalgam mesial amd distal walls are divergent

Gamma 2 phase of amnalgam weakest and least corrosion resistan Gamma- strongest and most corrosion resistant 2 sodium nitride used as antirust agent

Mercury is not relased in electrochemical corrosion

Amalgam (28-45PPM) AND COMPSOITE 25 ppm similar coefficient of

thermal expansion

Threshold limit value value 0.05 mg of mercury per cubic meter of air is

the vapour lever to which average worker is exposed

Particles of amalgam dental alloy is 15-35 microns

The alloys containing zinc equal or less than 0.01% are zinc free or non

zinc alloys

According to ADA specification 1 amalgam should not expand /contract

more than 20 micrometer at 37c between 5 minutes to 24 hours after

beginning of triuration

Compressive strength of amalgam should be minimum 310MPA its

tensile strength is between 48-70 MPA

Creep value for low copper 0/8% -*% and high copper is 0.4 to/1%

Penetration corrosion doesnot occur in high amalgam

POLISHING should be done 24 hours after condenstaion

50 MICROMETERS of mercury per week is maximum level of occupation

exposure

Mercury in marginal areas of restoration is 2-3 % higher than in centre

After amalgam polishing the outermost surface has crystal arranged

flattened to tooth surface This is known as layer of beilby

Q149.In a class V cavity preparation M-D walls depends on a. Direction of enamel rods

b. Contours of gingiva

c. Size of carious lesion d. Location of contact area

Ans A. Direction of enamel rods

Page 64: Principles and Practice of Ophthalmology' pg 2875 2013 Q1.All of the ... New Anti-Anginal Drugs Fasudil • Rho kinase inhibitor ... Q7. Which is the most anticoagulant of choice for

―The dir ecti o n of me si al a nd dis tal w all s f o ll o ws th e di rec tio n of e na mel ro ds‖

Class V

Placement of external walls depends upon- Extent of caries

Primary determinant of the outline form is the extension of caries.

External shape is related to the contour of the marginal gingiva

Axial wall is slightly deeper at the Incisal wall, where there is more enamel (1-1.25mm) than at the gingival wall, where there may be little or no enamel (0.75-1mm)

Direction of mesial and distal walls depends upon- Direction of enamel rods.

Shape of the cavity- Contour of gingiva.

The axial wall is convex mesiodistally following the external contour of the tooth

Direct filling gold is ideal restorative material for Class V cavities.

“Sturdvent 4Th edition Page 756

Q150.A periapical lesion was discovered 1.5 years after RCT on maxillary central incisor. Apical

curettage and biopsy showed lesion to be apical cyst .2 year later lesion is large than it was before surgery.Which is most likely cause of continued failure?

a)Leaking from main canals

b)Unobturated accessory canals c)Apex was not resected

d. Actinomycosis Infection

Ans A ,(Leaking from main canals)

Q151. In treating a lateral incisor with periapical abscess and draining sinus,the

treatment of choice for drainage sinus is?

a) No treatment

b) Curretage the sinus tract

c) Cautherization d) Irrigation with sodium hypochloride

Ans A. No treatment

―The sinus tract ultimately heals by granulation tissue after endodontic therapy by itself.

In this case after drainage of periapical abscess by endodontic therapy sinus tract will heal by

its own

Q152.The prime action of topical fluoride is a

a. Conversion of hydroxyapetite to fluorapetite

b. Decrease in the plaque bacteria

c. Form a reservoir in saliva

d. Improve morphology of teeth Ans a. Conversion of hydroxyapetite to fluorapetite

Page 65: Principles and Practice of Ophthalmology' pg 2875 2013 Q1.All of the ... New Anti-Anginal Drugs Fasudil • Rho kinase inhibitor ... Q7. Which is the most anticoagulant of choice for

TOPICAL FLUORIDES

The term topical fluoride therapy refers to the use of systems containing relatively large

concentrations of fluoride that are applied locally or topically, to the erupted tooth surface to

prevent the formation of dental caries.

Rationale for the Use of Topical Fluorides

Water fluoridation is the most effective, efficient and economical measure for the

prevention of dental caries. But fluoridated water is available to only a few percent of the population, hence alternative methods for the fluoride therapy is required.

At the time of tooth eruption, the enamel is not yet completely calcified and undergoes a

post eruptive period, approximately 2 years, during which enamel calcification continues [enamel maturation period]. During this period fluoride deposition takes place in the superficial layer of enamel.

This fluoride is derived from saliva and also from exposure of teeth to fluoride- containing water and food. Thus most of fluoride incorporation into enamel occurs during the

preeruptive period (by systemic fluoride) and the post eruptive period of enamel maturation. As immediately after eruption, the enamel is porous and immature it acquires fluoride rapidly .

Application of topical fluorides immediately after eruption hastens fluoride uptake and

makes enamel more resistant to dental caries.

ADVANTAGES AND DISADVANTAGES OF

TOPICAL FLUORIDES

Advantages

1. Does not cause fluorosis.

2. Cariostatic for people of all ages.

3. Available only to people who desire it.

4. Easy to use.

Disadvantages

1. Person must remember to use.

2. Per capita cost is high compared to water fluoridation.

3. More concentrated professional use products can cause short-term side effects like nausea

immediately after use. Not all fluoride agents and treatments are equal.

The efficacy of topical fluoride depends on:

a. The concentration of fluoride used.

Page 66: Principles and Practice of Ophthalmology' pg 2875 2013 Q1.All of the ... New Anti-Anginal Drugs Fasudil • Rho kinase inhibitor ... Q7. Which is the most anticoagulant of choice for

b. The frequency with which it is applied and the duration of application.

c. The specific fluoride compound used.

MECHANISM OF ACTION

The presence of elevated concentration of fluoride in enamel surface makes tooth surface

more resistant to development of dental caries. Fluoride ions when substituted into the

hydroxyapatite crystals fit more perfectly than do hydroxyl ions. Also the greater bonding potential of fluoride makes the apatite crystals more compact and more stable, thereby

more resistant to the acid dissolution. When concentrated topical fluoride agent reacts with enamel there is formation of calcium

fluoride: Most topical fluoride agents have a fluoride ion concentration of between

10,000–20,000 ppm which leads to the formation of calcium fluoride and eventually Fluor hydroxyapatite

Commonly used topical fluoride agents include Sodium fluoride, Sodium monofluorophosphate. Stannous fluoride and Amine fluoride

Points to Remember:-

Range of therapeutic fluoride concentrations used to prevent caries

Method/vehicle

Fluoride concentration (ppm

F)

Water supplies 0.7–1.2

Fluoridated salt

200–250

0.05% NaF Mouthrinse,

Once daily 230

0.2% NaF Mouthrinse,

Once weekly 920

Dentifrices, children

250–500

Dentifrices, adult Twice daily

1,000–1,500

1.1% NaF gels Once daily

5,000

Professionally applied solutions (2 % NaF)

9,200

Professionally applied solutions,

12,300

Professionally applied solutions (8% SnF2)

19,500

Professionally applied varnishes (5% NaF)

22,600

Page 67: Principles and Practice of Ophthalmology' pg 2875 2013 Q1.All of the ... New Anti-Anginal Drugs Fasudil • Rho kinase inhibitor ... Q7. Which is the most anticoagulant of choice for

Refer to AIIMS May 2013 paper for explanation

Q153. The common complication of the local anesthetic prilocaine is a. Aganulocytosis b. Hepatic Dysfunction

c. Methemoglobinemia

d. Loss of taste

Ans c. Methemoglobinemia

The systemic toxicity of prilocaine is comparatively low, however its metabolite, o-toluidine, is

known to cause methemoglobinemia. As methemoglobinemia reduces the amount of

hemoglobin that is available for oxygen transport, this side effect is potentially life-threatening.

Therefore dose limits for prilocaine should be strictly observed. Prilocaine is not recommended

for use in labor pains or infants.

Q154.The most common problem with blood transfusion is a. Hemolytic reaction

b. Metabolic acidosis c. Hypokalemia

d. Remission of disease

Ans a. (Hemolytic reaction)

"It is very significant that, by far, the most frequent complication of and the biggest risk to,

patients who receive blood transfusion is that of incompatible blood or blood products being

administered"-

"The most frequent serious complication of transfusion are acute hemolytic reactions due to

ABO blood group incompatibility and transfusion related acute lung injury (TRALI) –

ABC of Heart Failure by Russel, page 61.

Q155.Guddan, a 16year old boy is brought on Diwali night to the hospital with 60% burns.

Management includes: a. Packed red cells

Page 68: Principles and Practice of Ophthalmology' pg 2875 2013 Q1.All of the ... New Anti-Anginal Drugs Fasudil • Rho kinase inhibitor ... Q7. Which is the most anticoagulant of choice for

b. Whole blood c. Plasma

d. Fibrinogen

Ans c. (Plasma)

Fluid resuscitation should be started as soon as possible in adults with burns of 15% or

more of total body surface area, in case of children burns above 10% should be given

fluid.

Blood transfusion is required when burns involve more than 20% of full thickness or

40% of partial thickness of total body surface area.

Normal saline is isotonic and contains sodium and chloride in concentration plasma has. It is a solution for replacing gastrointestinal losses either by vomiting or by nasogastric

aspiration or through intestinal fistula. In dehydration there is loss of sodium from body .

5%dextrose supplies calories ,but not electrolytes.

The solution can only be used when the patient doesn‘t require any electrolyte but a

solution to replenish his blood volume along with some nutrition.

5% Dextrose Isotonic solution that supplies calories but not electrolyte

Used when the patient requires replenishment of his blood volume along with some nutrition,but no electrolyte

Particularly used in immediate post operative period when Na excretion

is considerably diminished by renal conservation

Isotonic

0.9% Sodium

chloride

It is isotonic and contains Na and Cl in the conc almost similar to that in

the plasma It should not be used in first 24 hrs after operation due to natural

sodium conservation

One important fact is that it contains a high concentration of Cl as compared to plasma and imposes an appreciable load of excess Cl on kidneys that cannot be readily excreted

Page 69: Principles and Practice of Ophthalmology' pg 2875 2013 Q1.All of the ... New Anti-Anginal Drugs Fasudil • Rho kinase inhibitor ... Q7. Which is the most anticoagulant of choice for

Ringer

Lactate Its main advantage is that it has almost similar electrolyte concentration

as ECF and the ph remains normal if infused on large quantities

This solution is best to be used in hypovolameic shock while waiting for blood

The chief disadvantage is its slight hypoosmolar with respect to Na

Darrow

Solution This is only solution which contains more potassium than available in

the plasma or ECF This is the best solution to combat hypokalemia

The rate of infusion should be slower than other solution to avoid

hyperkalemic state

Parenteral Fluid Constituents Use

Plasma, Albumin

4.5% Albumin, Na, K, Cl,

bicarbonate Severe burns

Dextrose 5% Low Na, and k concentration Post operative period

when sodium secretion

is reduced

Isotonic saline

0.9% High conc. Of sodium and

chloride Vomitings, gastric or

duodenal aspiration

Ringer‘s

lactate(hartmann‘s

solution)

Contain sodium, K, cl in the

conc. Similar to the plasma

Hypovolemic shock

“S - DAS – S U RGE RY T HI RD E DI TI ON ”

Q156.Dermatologic manifestation of drug allergy are managed by: a. Lorazepam b. Pirenzepine

c. Rantidine

d. Chlorpheniramine

Ans d. (Chlorpheniramine)

Page 70: Principles and Practice of Ophthalmology' pg 2875 2013 Q1.All of the ... New Anti-Anginal Drugs Fasudil • Rho kinase inhibitor ... Q7. Which is the most anticoagulant of choice for

" If the allergic reaction is confined to the skin, an antihistamine is administered either

intravenously or intramuscularly"

"Adrenaline should be followed by an antihistamine like Chlorpheniramine 10mg intravenously

and 20mg intravenously of hydrocortisone"

Serotonergic and norepinephrinergic effects

In addition to being an histamine H1 receptorantagonist, chlorphenamine has been shown to

work as a serotonin-norepinephrine reuptake inhibitor or SNRI.A similar antihistamine,

brompheniramine, led to the discovery of the SSRIzimelidine. Limited clinical evidence shows

that it is comparable to several antidepressant medications in its ability to inhibit the reuptake

of serotonin and also norepinephrine (noradrenaline).However, extensive clinical trials of its

psychiatric properties in humans have not been conducted. It inhibits serotonin reuptake less

than norepinephrine reuptake, however the literature is not consistent in this respect.

Adverse effects

The adverse effects include drowsiness, dizziness, confusion, constipation, anxiety, nausea,

blurred vision, restlessness, decreased coordination, dry mouth, shallow breathing,

hallucinations, irritability, problems with memory or concentration, ringing in the ears, and

trouble urinating.

"Generalized Anaphylaxis: Cutaneous rashes: oral administration or injection of

Chlorpheniramine, intramuscularly"

Text book of oral medicine by Ghom, page 724

Q157.Osteomyelitis of the mandible is caused by:

a. Nocardia asteroids

b. Borrelia vincentii

c. Staphylococcus aureus d. Actinomyces bovis

Ans c. (Staphylococcus aureus)

Page 71: Principles and Practice of Ophthalmology' pg 2875 2013 Q1.All of the ... New Anti-Anginal Drugs Fasudil • Rho kinase inhibitor ... Q7. Which is the most anticoagulant of choice for

"It is usually a polymicrobial infection. Different types of organisms may he cultured

from these lesions; the most common are Staphylococcus aureus and Staphylococcus albus,

and various streptococci. Anaerohes such as Bacteroides, Porphromonas or Prevotella spectes

also predominate"

Shafer's Textbook of Oral Pathology, 6th edition, page 492

Q158.To restrict the mandibular growth in a skeletal class three malocclusion, the minimum amount of force from achin cup should be: a. 1.5-2 gm/mm2 area of condylar head

b. 6-7 gm/mm2 area of condylar head

c. 15-20 gm/mm2 area of condylar head

d. 25-30 gm/mm2 area of condylar head

Ans c. 15-20 gm/mm2 area of condylar head

In classIII case to prevent the growth of mandible the force used with chin cap is 15-20

gm/mm2 condylar surface area.

Chin cup was introduced by Oppenheim. It delivers a force of 800 – 1200 ( 400-

600gm/side).The patient is asked to wear the appliance 12-14 hours/daily. It is used to restrict the downward and the forward growth of the mandible.

Types of chin cup Occipital

chin cup Vertical Chin cup

Occipital chin cap derives anchorage from occipital region most commonly

used,also used in cases of mild mandibular protrusive incsiors

Vertical Pull:Inclined with steep mandibular plane angle with anterior open bite,derives anchorage from parietal region

Philosophy of Chin Cup Therapy

Mandible grows by apposition of bone at the condyle and along its free posterior border.

Condyle is not a growth center and condylar growth is largely a response to translation of

surrounding tissues. This contemporary view offers a more optimistic view of the possibilities

for growth restraint of the mandible,

as with chin cup therapy.

Page 72: Principles and Practice of Ophthalmology' pg 2875 2013 Q1.All of the ... New Anti-Anginal Drugs Fasudil • Rho kinase inhibitor ... Q7. Which is the most anticoagulant of choice for

Magnitude of Force

Most authors recommend a force of 300-600 gm/side. Initially a lower force level (about 150 gm)

may be advised for the patient to get used to the appliance.

Effects of Chin cup

Extraoral force of the chin cup, directed against the mandibular growth. However, most human

studies have failed to conclusively prove that chin cup inhabits mandibular growth. However,

the following effects are seen.

a. Redirection of mandibular growth in a downward and backward direction.

b. Remodeling of the mandible and a decrease in mandibular plane angle and gonial angle

c. Lingual tipping of lower incisors.

d. Improvement in skeletal and soft tissue profile. Therefore, chin cup works well in patients

with reduced or normal lower anterior face height but is contradicted in long face patients.

According to TM Craber, ideal patients for chin cup therapy are those suffering from:

• A mild skeletal problem with the ability to bring the incisors.end-to-end or nearly so.

• Short vertical face height

• Normally positioned or protrusive, but not retrusive lower incisors.

Commercially Available

• Soft Elastic appliance

• Hickham-type appliance

• Unitek design

“Gr abb er LW . C hi n c up t her apy for ma n di bul ar pr og nat hi sm . Am J Orthod 1977;72:23-

4l .”

“CO NTE MPO RA RY O RT HOD ON TI CS WI LLIA M R. PR OFF I T, THI RD E DI TI ON”

Q159. The proximal fragment in a fracture of angle of mandible is displaced: a. Anterior and superiorly

b. Only posteriorly

Page 73: Principles and Practice of Ophthalmology' pg 2875 2013 Q1.All of the ... New Anti-Anginal Drugs Fasudil • Rho kinase inhibitor ... Q7. Which is the most anticoagulant of choice for

c. Only inferiorly d. Inferiorly and anteriorly

Ans a. Anterior and superiorly

Classification of mandibular fractures according to the Direction of Fracture and Favourability

for Treatment (Figs 31.5 and 31.6)

a. Horizontally favourable fracture.

b. Horizontally unfavourable fracture.

c. Vertically favourable fracture.

d. Vertically unfavourable fracture.

This classification is aimed toward the angle fractures. Here, the direction of fracture line

is important for resisting the muscle pull. When the muscle pull resists the displacement of the fragments, then the fracture line is

considered as favourable. If the muscle pull distracts the fragments away from each other, resulting in

displacement, then the fracture line is considered as unfavourable.

The elevator group of muscles exert an upward, forward and medial pull, while depressor group of muscles exert a downward and backward pull in an intact mandible. Whenever

there is a break in the continuity at the angle region, then these two muscle groups loose their coordinated movements and have independent action. In unilateral angle fracture,

posterior ramus fragment is the lesser fragment, while the body of the mandible bearing

the teeth becomes the greater fragment. The greater fragment‘s position is stabilized to

certain extent by the occlusion of the teeth, while posterior ramal fragment can show

displacement independently. Here, the direction of the fracture line is responsible for resisting the muscle pull. When

the mandible is viewed from the horizontal plane or studied by taking oblique lateral view mandible radiograph, then whether it is horizontally favourable or unfavorable line is

decided.

a. When the fracture line passes from the alveolar margin, downward and forward, then

upward displacement of the posterior fragment is prevented by physical obstruction caused by

the body of the mandible. Hence, such a fracture line is termed horizontally favourable.

b. If, on the other hand, the line of fracture passes downward and backward, then the upward

movement of the posterior fragment is unopposed. This type of fracture is termed horizontally

unfavourable.Sometimes the upward displacement can be prevented by the presence of a tooth

on the posterior fragment which comes into contact with maxillary tooth.

c. When the angle fracture is viewed from above, or from the occlusal surface (in the vertical

plane), then the buccolingual direction of the fracture line can be studied. Here the

displacement of the posterior fragment can be noticed in the medial direction due

to the spasm of medial pterygoid and mylohyoid muscle. Here the fracture line which passes

from the outer or buccal plate obliquely backward and lingually, will tend to resist the muscle

pull mentioned above and is thus termed a vertically favourable type of fracture.

Page 74: Principles and Practice of Ophthalmology' pg 2875 2013 Q1.All of the ... New Anti-Anginal Drugs Fasudil • Rho kinase inhibitor ... Q7. Which is the most anticoagulant of choice for

d. When the fracture line passes from the inner or lingual plate obliquely backward and

buccally, inward movement of the posterior fragment will take place as a result of the medial

pterygoid muscle pull. This type of fracture is termed vertically unfavourable.

Neelima Malik, 2nd edi, pg 383

Q160. The microorganism implicated in infective endocarditis is:

a. Streptococcus mitis b. Staphylococcus aureus

c. Streptococcus viridans

d. Staphylococcus pyogenes

Ans c. Streptococcus viridans

"Streptococci cause about 55% of cases of native valve endocarditis in patients who do not

abuse intravenous drugs. Streptococci viridians (most commonly strept.Sanguis, strept

mutans, strept mitis (formerly strept mitior) or strept. Milleri) account for 75% of these cases:

STrept viridans are normal inhabitants of oropharynx and generally are highly susceptible to

penicillin"

Harrison's Principles of Internal Medicine, 15th edition, page 785

Page 75: Principles and Practice of Ophthalmology' pg 2875 2013 Q1.All of the ... New Anti-Anginal Drugs Fasudil • Rho kinase inhibitor ... Q7. Which is the most anticoagulant of choice for

Q161.The case control studies following the distribution of smokers and non- smokers in patients of oral carcinoma and control. The Odds ratio (OR) of getting oral cancer in smokers

to that of non–smokers is:

Oral Cancer Control

Smoker 30 20

Non-smoker 20 30

a. 0.44 b. 1.0 c. 1.5 d. 2.25 Ans d. 2.25

Odds Ratio (OR)—It is a measure of the strength of association between the risk factor and

outcome.

Odd‘s Ratio= ad

bc

= 30 x 30

20 x 20

= 2.25

A Textbook of Public Health Dentistry by Marya, pg 16

Q162.Iron and folic acid supplement given to children is a form of: a. Primordial prevention

b. Specific protection

c. Health promotion d. Early intervention

Ans b. Specific protection

Modes of Intervention

Primary Prevention (Prepathogenesis)

Page 76: Principles and Practice of Ophthalmology' pg 2875 2013 Q1.All of the ... New Anti-Anginal Drugs Fasudil • Rho kinase inhibitor ... Q7. Which is the most anticoagulant of choice for

Primary preventive services are those that prevent the initiation of disease.

a. Health promotion: It is process of enabling people to increase control over and to improve

health. This can be achieved by

i. Health education; instruction on proper plaque removal, daily tooth brushing and flossing

ii. Environment modification such as safe water, control of insects and rodents.

iii. Nutritional interventions: improvement of nutrition in vulnerable group.

iv. Lifestyle and behavioural changes; which favor health

b. Specific protection: These are activities designed to protect against disease agents by

decreasing the susceptibility of the host or by establishing barrier against agents in

the environment. Methods include immunization, use of specific nutrition, avoidance of

allergens, protection from carcinogens, ingestion of optimally fluoridated water and

application of pit and fissure sealants.

Secondary Prevention

(Pathogenesis: Initial Stage of Pathogenesis)

It is defined as ―action which halts the progress of a disease at its incipient stage and prevents

complications.‖

These services intervene or prevent the progression and recurrence of disease.

a. Early diagnosis: WHO Expert Committee in 1973 defined early detection of health disorders

as ―the detection of disturbances of homeostatic and compensatory mechanism while

biochemical, morphological and functional changes are still reversible.‖

The earlier the disease is diagnosed and treated the better is its prognosis and helps to prevent

the occurrence of more cases.

Actions that detect and treat disease at an early stage thus hinder the progress of a disease

and prevent complications. i.e. intervention in early pathogenesis phase.

The methods (tools) employed for early diagnosis are:

1. Screening for sub-clinical disease, either in screening surveys or in periodic medical

examinations.

2. Case finding (individual and community).

Page 77: Principles and Practice of Ophthalmology' pg 2875 2013 Q1.All of the ... New Anti-Anginal Drugs Fasudil • Rho kinase inhibitor ... Q7. Which is the most anticoagulant of choice for

b. Prompt treatment:Secondary prevention attempts to arrest the disease process, restore

health by seeking out unrecognized disease and treating it before irreversible

pathological changes take place, and reverse communicability of infectious diseases.

Tertiary Prevention

(Pathogenesis: Late Stage of Pathogenesis)

Actions taken when the disease process has advanced beyond its early stages i.e.

intervention in late pathogenesis phase.

It is defined as ―all the measures available to reduce or limit impairments and disabilities, and to promote the patients‘ adjustment to irremediable conditions.‖

Intervention that should be accomplished in the stage of tertiary prevention is disability limitation, and rehabilitation.

The aim of tertiary prevention is to limit disability and prevent further complications or death.

a. Disability limitation

The objective of this intervention is to prevent or stop the transition of the disease process from

impairment to handicap. The sequence is as follows:

• Disease • Impairment • Disability • Handicap

i. Impairment: It is ―any loss or abnormality of psychological, physiological or anatomical

structure or function.‖

ii. Disability: It is ―any restriction or lack of ability to perform an activity in the manner or

within the range considered normal for the human being.‖

iii. Handicap: It is termed as ―a disadvantage for a given individual, resulting from an

impairment or disability that limits or prevents the fulfillment of a role in the community that

is normal (depending on age, sex, and social and cultural factors) for that individual.‖

b. Rehabilitation: It is defined as ―the combined and coordinated use of medical, social,

educational, and vocational measures for training and retraining the individual to the highest

possible level of functional ability.‖ It is a measure to train the disable individuals to reach the

highest level of functional ability by using combined coordinated medical, social, vocational,

psychological and educational measures.

Rehabilitation comprises:

1. Medical rehabilitation: restoration of function or physical loss.

2. Educational rehabilitation: change of educational methods.

Page 78: Principles and Practice of Ophthalmology' pg 2875 2013 Q1.All of the ... New Anti-Anginal Drugs Fasudil • Rho kinase inhibitor ... Q7. Which is the most anticoagulant of choice for

3. Vocational (occupational) rehabilitation: restoration of the capacity to earn a livelihood.

4. Social rehabilitation: restoration of family and social relationships.

5. Psychological rehabilitation: restoration of personal confidence

Examples of rehabilitation:

1. Special schools for blind pupils.

2. Provision of aids for crippled.

3. Reconstructive surgery for leprotics.

4. Modification of life for tuberculous or cardiac patients.

A Textbook of Public Health Dentistry by Marya, pg 8

Q163.In pencillin allergy acts as a a. Hapten

b. Carrier

c. Super antigen

d. Toxin Ans a. (Hapten)

"Hypersensitivty: This is the most important adverse effect of the penicillins. The major

antigenic determinant of penicillin hypersensitivity is its metabolite, penicilloic acid, which

reacts with proteins and serves as a hapten to cause an immune reaction

"Hapten that binds to a tissue protein and becomes antigenic because of the size of complex.

An IgE response to hapten epitopes can then lead to anaphylactic-type hypersensitivity if the

epitope is again encountered. A penicillin degradation product has this property"

Lippincott Ilustrated reviews Pharmacology, page 364

Q164.Enzyme that catalyses the transfer of a molecule group one molecule to another is:

a. Oxideases

b. Peptideases c. Transferases

d. Lipases

Page 79: Principles and Practice of Ophthalmology' pg 2875 2013 Q1.All of the ... New Anti-Anginal Drugs Fasudil • Rho kinase inhibitor ... Q7. Which is the most anticoagulant of choice for

Ans c. (Transferases)

"Transferases: Enzymes that catalyze the transfer of functional groups"

Biochemsitry by U. Satayanarayana, 2nd edition, page 86

Q165.The artery most commonly involved in stroke is the:

a. Lenticulostriate b. Anterior cerebral

c. Posterior cerebral d. Middle meningeal

Ans a. (Lenticulostriate)

(Direct pick from NBD Part-I, Anatomic Science, 0-2, Dec. 1985)

Q166.The primary germ layers develop during:

a. Pre-somite period

b. Somite period

c. Fetal Period

d. Post-somite period Ans a. (Pre-somite period)

"During the presomite period the primary germ layers of the embryo and the embryonic adnexa

(fetal membranes) are formed in the inner cell mass"

Craniofacial development by Geoffrey H. Sperber, page 18

Q167.Maxillary marrow contains

a. All fatty marrow

b. Red marrow c. Fibrous marrow

d. Fatty and fibrous marrow Ans a. (All fatty marrow)

"The bones of the face, such as maxilla and zygoma, contain completely fatty narrow and have

no hematopoietic marrow"

Page 80: Principles and Practice of Ophthalmology' pg 2875 2013 Q1.All of the ... New Anti-Anginal Drugs Fasudil • Rho kinase inhibitor ... Q7. Which is the most anticoagulant of choice for

Kyushu journal of Medical sciences: Vol-11

Q168.Epiphora is: a. Cerebral tumor b. Leak of cerebrospinal fluid through the nose due to fracture of anterior fossa

c. Abnormal overflow of tears following obstruction to the nasolacrimal duct

d. Eversion of the lower eyelid

Ans c. (Abnormal overflow of tears following obstruction of nasolacrimal duct)

"Epiphora: An abnormal overflow of teras down the cheek, mainly due to stricture of the

lacrimal passages; called also illacrimation"

"Another complication of sinus surgery affecting te eye is nasolacrimal duct injury.

Postoperative, the patients presents with epiphora or learning. The nasolacrimal duct courses

anterior to the natural ostium of the maxillary sinus and can be injured when te antrostomy is

enlarged anteriorly"

Peterson's Principles of Oral and Maxilofacial Surgery, 2nd edition, page 309

Points to Remember:-

Malunion of maxillary fractures can obstruct the nasolacrimal ducts. This obstruction

causes epipora and may lead to episodes of dacryocystitis.

Persistent tearing that dos not decrease after 3 weeks may need to be addressed by a

dacryocystorhinostomy

Q169.Sensory supply of the palate is through all of the following except:

a. Facial nerve

b. Hypoglossal nerve c. Glossopharyngeal nerve

d. Maxillary division of trigeminal nerve Ans b. (Hypoglossal)

Page 81: Principles and Practice of Ophthalmology' pg 2875 2013 Q1.All of the ... New Anti-Anginal Drugs Fasudil • Rho kinase inhibitor ... Q7. Which is the most anticoagulant of choice for

General sensory nerves of Palate

o Middle and posterior (lesser palatine) nerves, which are branches of maxillary nerve

(through the pterygopalatine ganglion)

o From the glossopharyngeal nerve Special sensory (gustatory) nerve)

o Carrying taste sensations from the oral surface are contained in the esser palatine

nerves. The fibers travel through the greater petrosal nerve to the geniculate

ganglion of the Facial nerve and from there to the nucleus of the solitary tract.

BD Chaurasia, 3rd edition, page 180

Q170.Calcium ions trigger muscles contraction by binding to:

a. Actin

b. Myosin c. Troponin

d. Tropomysin

Ans c. (Troponin)

―Ca2+ initiates contraction by binding to troponin C. In resting muscle, troponin I is tightly

bound to actin and tropomyosin covers the sites where myosin heads bind to actin… When te

Ca2+ released by the action potential binds to troponin C, the binding of troponin 1 to acting is

presumably weakened, and this permits the tropomyosin to move laterally. This movement

uncovers the binding site for myosin heads. ATP splits and contraction occurs" (Review

ofMedical Physiology by Ganong, 22nd edition, chapter 3)

"When Calcium ions combine with troponin C each molecule of which can bind strongly with

upto four calcium ions, the troponin complex supposedly undergoes a conformational change

that in some way tugs on tropomyosin molecule and moves it deeper into the groove between

the two actin strands. This uncovers the active site of actin and contration proceeds"

Text book of medical physiology by Gyuton, 11th edition, page 74

Points to Remember:

Page 82: Principles and Practice of Ophthalmology' pg 2875 2013 Q1.All of the ... New Anti-Anginal Drugs Fasudil • Rho kinase inhibitor ... Q7. Which is the most anticoagulant of choice for

Sequence of events in Contraction and Relaxation:

Page 83: Principles and Practice of Ophthalmology' pg 2875 2013 Q1.All of the ... New Anti-Anginal Drugs Fasudil • Rho kinase inhibitor ... Q7. Which is the most anticoagulant of choice for

Events during contraction Events during relaxation

Discharge of motor neuron Ca2+ pumped back into sarcoplasmic

reticulum

Release of Ca2+ from troponin

Release of transmitter (acetylcholine) at

motor end plate Cessation of interaction between actin

and myosin

Binding of ACh to nicotinic ACh receptors

Increased Na & K conductance

Generation of end-plate potential

Inward spread of depolarization along T- tubules

Release of Ca2+ from terminal cisterns of sarcoplasmic reticulum & diffusion to thick and thin filaments

Binding of Ca2+ to troponin C, uncovering myosin-binding sites on actin

Formation of cross-linkages between actin & myosin and sliding of thin on thick filaments, producing movements

Troponin subunits:

o Troponin I: Has strong affinity for action

o Troponin T: Has strong affinity for tropomyosin

o Troponin C: For calcium ions

In place of troponin, smooth muscles contain Calmodulin

Skeletal Muscle: Most of calcium for contraction is from sarcoplasmic reticulum

(intracellular)

Page 84: Principles and Practice of Ophthalmology' pg 2875 2013 Q1.All of the ... New Anti-Anginal Drugs Fasudil • Rho kinase inhibitor ... Q7. Which is the most anticoagulant of choice for

Smooth muscle: Most of the calcium for contraction is extracellular because it has less

developed sarcoplasmic reticulum.

Q171.Which of the following is present in the plasma but absent in serum

a. Albumin b. Globulin

c. Lecithin

d. Fibrinogen

Ans d. (Fibrinogen)

"Within a few minutes after a clot is formed. It begins to contract and usually express most of

the fluid from the clot within 20 to 60 minutes. The fluid expresses is called serum because all

its fibrinogen and most of other clotting factors have been removed; in this way, serum differs

from plasma. Serum cannot clot because it lacks these factors"

"Serum has essentially the same composition as plasma except that its fibrinogen and clottin

factors II, V and VIII have been removed and it has higher serotonin content because of

breakdown of platelets during clotting:

Ganong's Review of Medical Physiology, 22nd edition

Q172.Which of the following functions of bone is most prominent: a. Calcium reservoir

b. Sodium reservoir c. Phosphate source

d. Buffering

Ans a. (Calcium reservoir)

"Bone forms the strong and rigid endoskeleton to which skeletal muscles is attached to permit

movement. It also acts as a calcium reservoir and is important in calcium homeostasis"

Wheater's functional Histology, 5th edition, page 189

Q173.Significant decrease in serum calcium will cause: a. Hyperirritability of nerves and muscles

b. Positive ionotropic and chronotropic effect on the heart

c. Sudden increase in the thyroid and parathyroid

Page 85: Principles and Practice of Ophthalmology' pg 2875 2013 Q1.All of the ... New Anti-Anginal Drugs Fasudil • Rho kinase inhibitor ... Q7. Which is the most anticoagulant of choice for

d. Hyperirritability of nerves and muscles Ans d. (Hyperirritability of nerve and muscle)

"A decrease in extracellular Ca2+ exerts a net excitatory effect on the nerve and muscle cells in

vivo. The result is hypocalcemic tetany"

"A decrease in the extracellular Ca2+ concentration increases the excitability of nerve and

muscle cells by decreasing the amount of depolarization necessary to initiate the changes in

Na+, K conductance that produces the action potential"

"Conversely an increase in Ca concentration stabilizes the membrane by decreasing

excitability" (Review of Medical Physiology by Ganong, 20th edition, page 379)

Tetany ordinarily occurs when the blood concentration of calcium falls below its normal

level of 9.4mg/ dl to 6mg/dl (35% below normal value)

Usually lethal at about 4mg/dl.

Decreasing the external Na+ concentration decreases the size of action potential but has

little effect on RMP (resting membrane potential)

Increasing the external K+ concentration decreases the RMP.

Review of Medical Physiology by Ganong, 22nd edition, chapter 21

Q174.Magnesium is needed for the activity of which enzyme: a. Phosphatase

b. Aldolase c. Dismutase

d. ATPase

Ans a. (Phosphatase)

"Magnesium is required in covalent modification of enzymes by phosphorylation,

dephosphorylation of seryl residues"

Page 86: Principles and Practice of Ophthalmology' pg 2875 2013 Q1.All of the ... New Anti-Anginal Drugs Fasudil • Rho kinase inhibitor ... Q7. Which is the most anticoagulant of choice for

"Magnesium as seen in above is required with enzymes kinase and phosphatase. As kinase is

not provided in the options, phosphatase is the answer

Mg is also erquired in:

Kinase Peptidase Adenyl cyclase

Ribonuclease Ca Carboxylase Transketolase

Harper's Biochemistry, 25th edition, page 119; Chatterjea Shinde, 4th edition, page 557

Q175.The following host tissue responses can be seen in acute inflammation except: a. Exudation

b. Vasodilation

c. Margination d. Granuloma formation

Ans d. (Granuloma formation)

"Granulomatous inflammations is a distinctive pattern of chronic inflammation characterized

by aggregate of activated macrophages that assume an epitheloid appearance"

Hence granuloma formation is a feature of Chronic inflammation.

Robbin's Basic Pathology, 8th edition, page 56

Q176.If both the parents have cleft lip and cleft palate, the probability of first child having the

same condition is: a. 1:5 (2%)

b. 1:25 (4%) c. 1:10 (10%)

d. 3:5 (60%)

Ans d. 3:5 (60%)

National Board, Oral Surgery

Refer to AIIMS May 2012 paper for explanation

Q177. Toxoids are prepared by treating the toxins with:

a. Formaldehyde

Page 87: Principles and Practice of Ophthalmology' pg 2875 2013 Q1.All of the ... New Anti-Anginal Drugs Fasudil • Rho kinase inhibitor ... Q7. Which is the most anticoagulant of choice for

b. Pepsin c. Papain

d. Trypsin

Ans a. (Formaldehyde)

"Fortunately, exotoxin polypeptides are good antigens and induce the synthesis of protective

antibodies called antitoxins, useful inprevention or treatment of diseases such as tetanus. The

toxicity of the polypeptides can be neutralized when treated with formaldehyde (or acid or heat)

and these toxoids are used in protective vaccines, because they retain antigenicity"

Essential Microbiology for Dentistry by Samaranayake, 3rd edition, page 40

Q178.Adenosine deaminase (enzyme) deficiency is associated with:

A. Severe combined immunodeficiency (SCID)

B. X-linked agammaglobulinemia

C. Transient hypogarnmaglobulinemia of infancy

D. Chronic granulomatous disease

Correct answer : A. Severe combined immunodeficiency (SCID)

Q179)Which of the following is not a neuroparasite?

A. Taenia solium

B. Acanthamoeba

C. Naegleria

D. Trichinella spiralis

Correct answer : D. Trichinella spiralis

Trichinella spiralis is seen in striated muscles.

Q180)The drug used in ATRIDOX local delivery system is: A. Penicillin B. Metronidazole

C. Doxycycline

D. Chlorhexidine

Ans C. Doxycycline

Page 88: Principles and Practice of Ophthalmology' pg 2875 2013 Q1.All of the ... New Anti-Anginal Drugs Fasudil • Rho kinase inhibitor ... Q7. Which is the most anticoagulant of choice for

Ref:- Carranza 10th edi, pg 809

Q181.The direction used in Naber’s probe: A. Lateral B. Horizontal

C. Vertical

D. Vertical & horizontal

Ans B. Horizontal

In multirooted teeth the possibilty of furcation involvement should be carefully

explored.The use of specially designed probes ( Nabers probe) allows an easier and

more accurate exploration of the horizontal component of furcation leisons

“Carranza Text Book of Periodontology”

Q182.The following features is observed in aging periodontal ligament: A. Increased in cell size B. Increase in cell number

C. Scalloping between alveolar bone and cementum D. Increase in cellular content

Ans C. Scalloping between alveolar bone and cementum

Page 89: Principles and Practice of Ophthalmology' pg 2875 2013 Q1.All of the ... New Anti-Anginal Drugs Fasudil • Rho kinase inhibitor ... Q7. Which is the most anticoagulant of choice for

With the increase in age:-

Keratinization of gingiva decreases

Width of attached gingiva increases

Number of elastic fibers in the periodontal ligament increases

Number of collegen fibers decreases.

Effect of aging on the periodontium

Gingival epithelium

Thinning and decreased keratinization Increase in epithelial permeability

Flattening of rete pegs

Width of attached gingiva increases

Connective tissue

Collegen content decreases Rate of collegen synthesis increases

Periodontal Ligament Number of fibroblast decreases Decreased organic matrix formation

Cementum

Increase in cemental width Increase in width is greater apically and lingually

Accumulation of resorption bays explains the finding of increasing

surface irregularity.

Ref:- Carranza 10th edi, pg93-96

Q183. The shape of the dentinal caries in spread of caries is: A. Always V-shaped with base towards DEJ B. Always V-shaped with base towards pulp

C. Depends on type of lesion

D. None of above

Ans A. Always V-shaped with base towards DEJ

Page 90: Principles and Practice of Ophthalmology' pg 2875 2013 Q1.All of the ... New Anti-Anginal Drugs Fasudil • Rho kinase inhibitor ... Q7. Which is the most anticoagulant of choice for

Clinical and Histologic Characteristics of Dentinal Caries, Acid Levels, and

Reparative Responses. Progression of caries in dentin is different from progressionin the overlying

enamel because of the structural differences of dentin

Dentin contains much less mineral and possesses microscopic tubules that

provide a pathway for the ingress of acids and egress of mineral.

The dentinoenamel junction has the least resistance to caries attack and allows

rapid lateral spreading once caries has penetrated the enamel.

Because of these characteristics, dentinal caries is V-shaped in cross-

sectionwith a wide base at the DEJ and the apex directed pulpally.

Caries advances more rapidly in dentin than in enamel because dentin provides

much less resistance to acid attack because of less mineralized content.

Caries produces a variety of responses in dentin, including pain,

demineralization and remineralization.

Q184.The most definitive diagnosis of acute pulpal degeneration is given by:

A. Electric pulp testing

B. History of unprovoked spontaneous pain

C. Thermal testing

D. Radiograph

Ans B. History of unprovoked spontaneous pain

Teeth affected by either acute reversible pulpitis or irreversible pulpitis are abnormally

responsive to cold and have many similar symptoms.

It is therefore essential that they can be distinguished from one another because the

emergency procedure for each is different.

If a patient describes pain that lasts for minutes to hours or occurs when bending over,

most likely that patient will require pulpectomy of the affected tooth rather than palliative

therapy for relief of the painful symptoms.

Ref: Grossman,11th

edi, pg 19

Q185.True regarding energy required during polymerization of composite in complete thickness

is:

A. 600 mW/cm2

B. 175 mW/cm2

C. 150 mW/cm2

D. 100 mW/cm2

Ans A. 600 mW/cm2

Page 91: Principles and Practice of Ophthalmology' pg 2875 2013 Q1.All of the ... New Anti-Anginal Drugs Fasudil • Rho kinase inhibitor ... Q7. Which is the most anticoagulant of choice for

Types of Lamps Used for Photoinitiator Curing

Four types of lamps may be used for photoinitiation of the polymerization process. The

following lists these lamps in order of lowest intensity to highest intensity.

1. LED lamps. Using a solid-state, electronic process, these light sources emit radiation

only in the blue part of the visible spectrum between 440 and 480nm, and they do not

require filters. LEDs require low wattage, can be battery powered, generate no heat, and

are quiet because a cooling fan is not needed. Although they produce the lowest intensity

radiation, new technology is rapidly overcoming this limitation.

2. QTH lamps. QTH lamps have a quartz bulb with a tungsten filament that irradiates

both UV and white light that must be filtered to remove heat and all wavelengths

except those in the violet-blue range( ~ 4 0 0-500 nm).The intensity of the bulb

diminishes with use, so a calibration meter is required to measure the output intensity.

3. PAC lamps. PAC lamps use a xenon gas that is ionized to produce a plasma. The

high-intensity white light is filtered to remove heat and to allow blue light ( ~ 4 0 0to 500

nm) to be emitted.

4. Argon laser lamps. Argon laser lamps have the highest intensity and emit at

a single wavelength. Lamps currently available emit -490 nm.

For maximum curing, which is about 50% to 60% monomer conversion , a

radiant energy influx of approximately 16,000 millijoules/cm2 (16 joules/cm2) is

required for a 2-mm-thick layer of resin.

This can be delivered by a 40-sec exposure to a lamp emitting 400 mW/cm2

(40 sec into 400 mw/cm2 = 16,000 mJ/cm2 or~ 1 6 J/cm2).

The same result can be produced by a 20-sec exposure at 800 mW/cm2, or an

exposure of -13 sec with a 1200-mW/cm2 lamp.

Thus increasing the power density of the lamp increases the rate and degree of

cure.

Ref: Skinners 11th

edi, pg412

Q186.All are regarding the polymerization pf composite using Tungsten Halogen light,

except: A. Wavelength less than 500nm B. Difficult to polymerize inaccessible areas like porclein inlays

C. Efficiency of light decreases with use

D. Irradiance range from 400 to 800 mw/cm2

Ans B. Difficult to polymerize inaccessible areas like porclein inlays

Page 92: Principles and Practice of Ophthalmology' pg 2875 2013 Q1.All of the ... New Anti-Anginal Drugs Fasudil • Rho kinase inhibitor ... Q7. Which is the most anticoagulant of choice for

Generally, QTH curing lights functioning in the normal range have outputs

from400 to 800 mW/cm2.

A good rule of thumb isthat the minimum output should never drop below 300 mW/cm2.

The intensity of bulb diminishes with use, so a calibration meter is required.

Peak wavelength ranges from 450 to 490 mw/cm2. A radiometer is designed to measure the photon level per unit time through a

standard 11 mm diameter window.

Therefore smaller or larger curing unit tips cannot be effectively tested. Light

energy entering into a fiber-optic bundle is diffused or concentrated depending

onwhether the curing unit tip is larger or smaller, respectively.

Shifting from a standard 11-mm diameter tip to a small 3 mm diameter has the

effect of increasing the light output eightfold. This increases the chance that

heat produced in the curing procedure will raise the temperature of the

restoration andsurrounding dentin to much more dangerous levels.

Increases in pulpal temperatures of more than 5° to 8° C easily cause cell death.

Ref: Skinners 10th

edi, pg377

Q187.The following is true about Glass ionomer cement: A. There is no cross linking between sodium and fluoride ions B. The liquid should not be refrigerated

C. It is strongly contraindicated in mouth breathers

D. Its strength is more than that of silicate

Ans A. There is no cross linking between sodium and fluoride ions

Page 93: Principles and Practice of Ophthalmology' pg 2875 2013 Q1.All of the ... New Anti-Anginal Drugs Fasudil • Rho kinase inhibitor ... Q7. Which is the most anticoagulant of choice for

GLASS lONOMER CEMENT

Chemistry of Setting of GIC

When the powder and liquid are mixed to form a paste, the acid etches the surface

of the glass particles and calcium, aluminum, sodium, and fluorine ions are leached into the aqueous medium.

The polyacrylic acid chains are cross-linked by the calcium ions that are replaced by

aluminum ions within the next 24 hr.

Sodium and fluorine ions do not participate in the cross-linking of the cement.

Some of the sodium may replace the hydrogen ions of the carboxylic groups, whereas

the remaining ions are dispersed uniformly within the set cement along with fluorine

ions.

Ihe cross-linked phase becomes hydrated over time with the same water used for

mixing. This process is called maturation.

'l'he unreacted portion of glass particles are sheathed by a silica gel that develops

during removal of cations from the surface of the particles. Thus the set cement

consists of an agglomeration of unreacted powder particles surrounded by a silica gel

in an amorphous matrix of hydrated calcium and aluminum polysalts.

Water plays a critical role in the settlng of G1C. It serves as the reaction medium

initially and then slowly hydrates the cross-linked matrix, thereby yielding a stable

gel structure that is stronger and less susceptible to moisture.

Ref: Skinners 11th edi, pg474

Q188.The following is true about lathe cut amalgam: A. Is available only as spherical alloys

Page 94: Principles and Practice of Ophthalmology' pg 2875 2013 Q1.All of the ... New Anti-Anginal Drugs Fasudil • Rho kinase inhibitor ... Q7. Which is the most anticoagulant of choice for

B. 1 hour strength is comparable to that of admix and unicompositional alloy is highest

C. Expansion is increased

D. Creep is less

Ans B. 1 hour strength is comparable to that of admix and unicompositional alloy is

highest

Ref: Sturdevant 4th

edi,Pg 156

Ref: Skinners 11th edi, pg512 Q189.Reciprocal arm, true is:

A. Rigid and is placed in middle 3rd

B. Flexible and is placed in middle 3rd

C. Rigid and is placed in occlusal 3rd

D.

Flexible and is placed in occlusal 3rd

Ans A. Rigid and is placed in middle 3rd

Page 95: Principles and Practice of Ophthalmology' pg 2875 2013 Q1.All of the ... New Anti-Anginal Drugs Fasudil • Rho kinase inhibitor ... Q7. Which is the most anticoagulant of choice for

pg86

Q190. The arrangement of artificial teeth: A. Is symmetrical to the right B. Is symmetrical to both sides

C. Depends on ridge shape and condition

D. Is assymetrically symmetrical

Ans C. Depends on ridge shape and condition

According to Zarb and Bolender

“ The buccolingual position of the posterior teeth and the arch are determined

anteriorly by the position of the canine and posteriorly by the basal seat and

the location of the retromolar pads.”

According to Rahn & Heartwell, 5th

edi, pg 332

“The horizontal positions of the posterior and the anterior teeth in dentulous

situations usually follow the form and the shape of the arch.”

Q192.The arrangement of artificial teeth: A. Is symmetrical to the right B. Is symmetrical to both sides

C. Is symmetrical to left side

D. Is assymetrically symmetrical

Ans D. Is assymetrically symmetrical

A ccordi n g to B ou ch er’s 9yh edi, pg 82

“If the teeth on the both sides of the arches were inclined parallel to each other,

they would make a most unsatisfying appearing denture. There should be as

assymetrical symmetry in the arrangement of the teeth.”

Page 96: Principles and Practice of Ophthalmology' pg 2875 2013 Q1.All of the ... New Anti-Anginal Drugs Fasudil • Rho kinase inhibitor ... Q7. Which is the most anticoagulant of choice for

Q193.The following material is used for taking impression of flabby ridge: A. Alginate B. Impression plaster

C. Putty silicone

D. Heavy bone silicone

Ans A. Alginate

Flabby Ridge: Most frequently seen in the anterior region Bone become grossly resorbed to level of anterior nasal spine and is replaced by

fibrous tissue

Etiology:Condition is sometimes called as combination syndroe and is caused by

presence of lower natural teeth

Management: 2 APPROACH

1) Surgical removal of fibrous tissue is favoured in every case where the health of

patient allows.

This produces a firm ridge which is reduced in size

2) If a flabby ridge is retained,impression technique can be

a)A mucostatic impression technique with the arm of achieving a maximum retention

b)a mucodisplacing impression technique which compresses the flabby tissue in order

to try and obtain a maximum support from it.

Suggested approach to flabby ridge problem:

Preliminary impression are taken in a stock tray using alginate of lower viscosity

Previous denture are left out for several days before working impressions for resolution of

inlammation,if any o denture bearing mucosa

Working impression is made in special tray using impression of low viscosity such as

alginate,impression plaster,and low viscosity silicone impresion

In case of alginate pressure on displaceable tissue can further reduced by using a tray

with perforation in the region covering flabby ridge.

So, a low viscosity materials such as algiante,impression plaster,and low viscosity

silicone can be used for flabby ridges,and not right viscosity materials such as

impression compound,or heavy /putty silicone as provided in the options.

Q194.. A patient in an excellent health wearing complete denture, complains of burning

sensation in anterior palate, may be due to

a. Nicotinamide deficiency b. Defective occlusal contact c. Failure to relieve incisive papilla d. Excessive thickness in rugae area Ans c. Failure to relieve incisive papilla

Page 97: Principles and Practice of Ophthalmology' pg 2875 2013 Q1.All of the ... New Anti-Anginal Drugs Fasudil • Rho kinase inhibitor ... Q7. Which is the most anticoagulant of choice for

Ref:- Boucher‟s 12th edi, pg 214, NBDE Prostho july,1978 q 17

Q195. Which of the following cements not be used to cement acrylic temporary crown

a. Zinc oxide eugenol b. Zinc phosphate c. Glass ionomer

d. Polycarboxylate

Ans c. Glass ionomer

Glass ionomer and polycarboxylate cements form chemically bonds with tooth structure. So they are

not used for cementation of temporary restorations. These should be cemented with a temporary

cement of moderate strength. Zinc oxide eugenol is the best choice.

Shillingburg 3rd edi, pg 233

Q196.. Brachytherapy is

a. Irradiation of tissues by radiopharmaceuticals b. Irradiation of tissues from a distance c. Irradiation of tissues from a distance of 3mm d. Irradiation of tissues by implants within the tissues Ans d. Irradiation of tissues by implants within the tissues

Page 98: Principles and Practice of Ophthalmology' pg 2875 2013 Q1.All of the ... New Anti-Anginal Drugs Fasudil • Rho kinase inhibitor ... Q7. Which is the most anticoagulant of choice for

Brachytherapy also known as internal radiotherapy, sealed source radiotherapy, curietherapy or endocurietherapy, is a form of radiotherapy where a radiation source is placed inside or next to the area requiring treatment.

Brachytherapy is commonly used as an effective treatment for cervical, prostate, breast, and skin cancer and can also be used to treat tumours in many other body sites.

Brachytherapy can be used alone or in combination with other therapies such as surgery, External Beam Radiotherapy (EBRT) and chemotherapy.

Brachytherapy contrasts with unsealed source radiotherapy in which a therapeutic radioisotope is injected into the body to chemically localize to the tissue which requires destruction.

It also contrasts to EBRT, in which high-energy x-rays (or occasionally gamma-rays from a radioisotope like cobalt-60) are directed at the tumour from outside the body.

Brachytherapy instead involves the precise placement of short-range radiation-sources (radioisotopes) directly at the site of the cancerous tumour.

These are enclosed in a protective capsule or wire which allows the ionizing radiation to escape to treat and kill surrounding tissue, but prevents the charge of radioisotope from

moving or dissolving in body fluids.

The capsule may be removed later, or (with some radioisotopes) it may be allowed to remain in place.A key feature of brachytherapy is that the irradiation only affects a very localized area around the radiation sources.

Exposure to radiation of healthy tissues further away from the sources is therefore reduced. In addition, if the patient moves or if there is any movement of the tumour within the body during treatment, the radiation sources retain their correct position in relation to the tumour.

These characteristics of brachytherapy provide advantages over EBRT - the tumour can be treated with very high doses of localised radiation, whilst reducing the probability of unnecessary damage to surrounding healthy tissues.

197. Dead bone is recognized on X-ray because

a. More radioopaque b. More radiolucent than normal c. Osteophytes grow around it d. It has soup bubble appearance Ans a. More radioopaque

Page 99: Principles and Practice of Ophthalmology' pg 2875 2013 Q1.All of the ... New Anti-Anginal Drugs Fasudil • Rho kinase inhibitor ... Q7. Which is the most anticoagulant of choice for

Radiographic findings

Radiographic changes occur only 3 weeks after initiation of OML process. It is generally accepted that 30-60% of mineralized portions of bone must be destroyed before

significant radiographic changes are noted.

In early stage, there is widening of marrow spaces, and enlargement of Volkmann‘s canals, which imparts a ― mo ttle d a ppe ar an ce ”. The granulation tissue between living and

dead bone produces irregular lines and zones of radiolucency. This results in characteristic ―moth-e ate n appe ar an ce ” of established OML.

In later stages, the cortex gets involved and due to ischaemia, an island of cortical bone becomes devitalized and becomes favourable for the precipitation of ionized calcium,

mobilized by the surrounding osteolytic process. The gradual resorption around periphery of infarcted area of bone separates it off as a

sequestrum.

Such a devitalized piece of bone appears sclerosed and becomes a foreign body which is called

as sequestrum. These segments appear highly calcified and therefore appear prominent. Large areas of bone destruction are seen as radiolucent areas. Subperiosteal new bone, the involucrum, can be seen as a fine linear opacity, or as a series of

laminated opacities, like an onion– skin, parallel to surface of cortex.

Where new bone is superimposed upon that of jaw, a delicate ―fingerprint‖ or ―orange-pe e l” appearance is seen.

Neelima malik 2nd edi, Pg 642

198. Radiographic density can be increased by

a. Decreasing the mA b. Decreasing the Kvp c. Increasing the object-film distance d. Decreasing the target-object distance Ans d. Decreasing the target-object distance

RADIOGRAPHIC DENSITY

When a film is exposed by an x-ray beam (or by light, in the case of screen-film combinations) and then

processed, the silver halide crystals in the emulsion that were struck by the photons are converted to grains of metallic silver.

These silver grains block the transmission of light from a viewbox and give the film its dark appearance.

The overall degree of darkening of an exposed film is referred to as radiographic density . This density can be measured as the optical density of an area of an x-ray film. Thus the measurement of film density also is a measure of the opacity of the film. With an optical density of 0, 100% of the light is transmitted; with a density of 1, 10% of the light is

transmitted; with a density of 2, 1% of the light is transmitted, and so on.

A plot of the relationship between film optical density and exposure is called a characteristic curve . A film is of greatest diagnostic value when the structures of interest are imaged on the relatively

straight portion of the graph, between 0.6 and 3.0 optical density units. The characteristic curves of films reveal much information about film contrast, speed, and latitude.

An unexposed film, when processed, shows some density. This is caused by the inherent density of the base and added tint and the development of unexposed

Page 100: Principles and Practice of Ophthalmology' pg 2875 2013 Q1.All of the ... New Anti-Anginal Drugs Fasudil • Rho kinase inhibitor ... Q7. Which is the most anticoagulant of choice for

silver halide crystals. This minimal density is called gross fog , or base plus fog.The optical density of

gross fog typically is 0.2 to 0.3. Radiographic density is influenced by exposure and the thickness and density of the subject.

Exposure

The overall film density depends on the number of photons absorbed by the film emulsion. Increasing

the milliamperage (mA), peak kilovoltage (kVp), or exposure time increases the number of photons reaching the film and thus increases the density of the radiograph.

Reducing the distance between the focal spot and film also increases film density.

Points to Remember:-

Subject Thickness

The thicker the subject, the more the beam is attenuated and the lighter the resultant image.

If exposure factors intended for adults are used on children or edentulous patients, the resultant films are dark because a smaller amount of absorbing tissue is in the path of the x-ray beam.

The dentist should vary exposure (either kVp or time) according to the patient ‘ s size to produce radiographs of optimal density.

Subject Density

The greater the density of a structure within the subject, the greater the attenuation of the x-ray

beam directed through that subject or area. In the oral cavity the relative densities of various natural structures, in order of decreasing

density, are enamel, dentin and cementum, bone, muscle, fat, and air. Metallic objects (e.g., restorations) are far denser than enamel and hence better absorbers.

RADIOGRAPHIC CONTRAST

Radiographic contrast defined as the difference in densities between light and dark regions on a

radiograph. Thus an image that shows both light areas and dark areas has high contrast . This also is

referred to as a short gray scale of contrastbecause few shades of gray are present between the

black and white images on the film. A radiographic image composed only of light gray and dark gray zones has low contrast , also

referred to as having a long gray scale of contrast.

The radiographic contrast of an image is the result of the interplay of subject contrast, film contrast, and scattered radiation.

Subject Contrast

Subject contrast is the range of characteristics of the subject that influences radiographic

contrast. It is influenced largely by the subject ‘ s thickness, density, and atomic number. The subject contrast of a patient ‘ s head and neck exposed in a lateral cephalometric view is

high.

Page 101: Principles and Practice of Ophthalmology' pg 2875 2013 Q1.All of the ... New Anti-Anginal Drugs Fasudil • Rho kinase inhibitor ... Q7. Which is the most anticoagulant of choice for

Film Contrast

Film contrast describes the capacity of radiographic films to display differences in subject

contrast, that is, variations in the intensity of the remnant beam. A high-contrast film reveals areas of small difference in subject contrast more clearly than

does a low-contrast film. Film contrast usually is measured as the average slope of the diagnostically useful portion of

the characteristic curve the greater the slope of the curve in this region, the greater the film contrast.

In this illustration, film A has a higher contrast than film B. When the slope of the curve in the useful range is greater than 1, the film exaggerates

subject contrast. This desirable feature, which is found in most diagnostic film, allows visualization of

structures that differ only slightly in density. For example, the remnant beam in the region of a tooth pulp chamber will be more intense (greater exposure) than the beam from the

surrounding enamel crown. A high-contrast film will show a greater contrast (difference in optical density) between these

structures than will a low-contrast film. Films used with intensifying screens typically have a slope in the range of 2 to 3.

White & Pharoah 6th edi, pg 60

Page 102: Principles and Practice of Ophthalmology' pg 2875 2013 Q1.All of the ... New Anti-Anginal Drugs Fasudil • Rho kinase inhibitor ... Q7. Which is the most anticoagulant of choice for

ORAL SURGERY

199. Polyglycolic acid sutures are:

a. Absorbable

b. Non-absorbable

c. Derived from the gut of sheep

Page 103: Principles and Practice of Ophthalmology' pg 2875 2013 Q1.All of the ... New Anti-Anginal Drugs Fasudil • Rho kinase inhibitor ... Q7. Which is the most anticoagulant of choice for

d. If braided cause more tissue reaction.

Ans a. Absorbable

200Which of following represents the correct patient position for left lateral

cephalogram?

a) Mid saggital plane should be 5 feet from tube

b) Right ear should be 5 feet from tube

c) Left ear should be 5 feet from tube

d) Left ear should be 5 feet from tube

Ans A. Mid saggital plane should be 5 feet from tube

“The distance between x ray source and the mid saggital plane of patient is fixed at 5 feet”